You are on page 1of 76

.

IASBABA’S AIPTS 2024 – TEST 4 (CURRENT AFFAIRS – JUNE 2023)

Q.1) Consider the following statements: b) Both Statement-I and Statement-II


are correct and Statement-II is not
1. The Delimitation Commission is the correct explanation' for
appointed by the Prime Minister of Statement-I
India and works in collaboration c) Statement I is correct but
with the Election Commission of Statement II is incorrect
India (ECI). d) Statement I is incorrect Statement
2. The Constitution of India mandates II is correct
that the allocation of seats in the
Lok Sabha should be based on the Q.3) Consider the following statements:
population of each state.
3. The 87th Amendment Act of 1. Graphene is used for stealth
2013 provided for the delimitation coatings and materials that reduce
of constituencies based on the radar signatures.
2011 census. 2. Graphene is an excellent candidate
for sensing chemical and biological
How many of the above statements are agents, and explosives.
correct? 3. Graphene finds applications in
aerospace, automotive, sports
a) Only one
equipment, and construction.
b) Only two
c) All three How many of the above statements are
d) None correct?

Q.2) Consider the following statements: a) Only one


b) Only two
Statement-I: c) All three
Evergreening loans is a practice d) None
of extending new or additional loans to a
borrower who is unable to repay the Q.4) Consider the following statements:
existing loans. 1. Sustainable Aviation Fuel (SAF) is
Statement-II: produced using domestically
Evergreening loans can create a false developed methods using cooking
impression of the asset quality and oil.
profitability of banks. 2. SAF has the greatest potential to
reduce CO2 emissions from
Which one of the following is correct in
International Aviation.
respect of the above statements?
3. SAF cannot be blended with
a) Both Statement-I and Statement-II traditional jet fuel.
are correct and Statement-II is the How many of the above statements are
correct explanation for Statement-I correct?
a) Only one
.
IASBABA’S AIPTS 2024 – TEST 4 (CURRENT AFFAIRS – JUNE 2023)

b) Only two modern slavery by 10 million


c) All three people since 2016.
d) None
How many of the above statements are
Q.5) Consider the following statements: correct?
a) Only one
Statement-I: b) Only two
The City Investments to Innovate, c) All three
Integrate and Sustain (CITIIS) 2.0 aims d) None
to promote a circular economy.
Statement-II: Q.7) Consider the following statements:
CITIIS, a sub-component of Smart Cities
Mission, is a joint program of the Ministry Statement-I:
of Housing and Urban Affairs, and Agence The World Health Assembly (WHA) is the
Francaise de Development (AFD). WHO’s decision-making body attended by
delegations from all of WHO’s member
Which one of the following is correct in states.
respect of the above statements? Statement-II:
The World Health Assembly (WHA) is held
a) Both Statement-I and Statement-II yearly at the headquarters of the World
are correct and Statement-II is the Health Organisation in New York.
correct explanation for Statement-I
b) Both Statement-I and Statement-II Which one of the following is correct in
are correct and Statement-II is not respect of the above statements?
the correct explanation for
Statement-I a) Both Statement-I and Statement-II
c) Statement I is correct but are correct and Statement-II is the
Statement II is incorrect correct explanation for Statement-I
d) Statement I is incorrect Statement b) Both Statement-I and Statement-II
II is correct are correct and Statement-II is not
the correct explanation' for
Q.6) Consider the following statements: Statement-I
c) Statement I is correct but
1. According to the Global Slavery Statement II is incorrect
Index, North Korea has the highest d) Statement I is incorrect Statement
prevalence of modern slavery. II is correct
2. According to the Global Slavery
Index, the United States has the Q.8) Consider the following statements:
lowest prevalence of modern 1. Article 299 of the Constitution
slavery. deals with the manner and form of
3. According to the Global Slavery contracts made by or on behalf of
Index 2023, there is a decrease in
.
IASBABA’S AIPTS 2024 – TEST 4 (CURRENT AFFAIRS – JUNE 2023)

the government of India or any 2. Purana Qila has the Qila-i-Kuhna


state government. mosque of Sher Shah Suri.
2. The Crown Proceedings Act of 3. In the recent excavations at Purana
1947 played a role in shaping Qila, a well from the Mauryan
Article 299. period was unearthed.
3. Mrs. Aliakutty Paul vs The State of How many of the above statements are
Kerala and Ors Case (1995) is correct?
related to Article 299. a) Only one
How many of the above statements are b) Only two
correct? c) All three
a) Only one d) None
b) Only two
c) All three Q.11) Consider the following statements
d) None about Mission Vatsalya:
1. It’s an umbrella scheme for widow
Q.9) Consider the following statements: protection services in the country.
Statement-I: 2. It is implemented by the Ministry of
Direct Seeded Rice (DSR) method Women and Child Development.
significantly reduces labor costs and eases 3. Its components include upscaling
the burden on farmers. institutional care and emergency
Statement-II: outreach services.
In this method, seeds are directly drilled Choose the correct code:
into the fields, eliminating the need for a) 1 and 2
nursery preparation and transplantation. b) 2 and 3
Which one of the following is correct in c) 1 and 3
respect of the above statements? d) 1, 2 and 3
a) Both Statement-I and Statement-II
are correct and Statement-II is the Q.12) Consider the following statements
correct explanation for Statement-I about the Lightweight Payments System:
b) Both Statement-I and Statement-II 1. It ensures uninterrupted digital
are correct and Statement-II is not payments during emergencies such
the correct explanation' for as natural calamities or war.
Statement-I 2. It is dependent on existing
c) Statement I is correct but payment technologies like Unified
Statement II is incorrect Payment Interface and National
d) Statement I is incorrect Statement Electronic Funds Transfer.
II is correct Choose the correct code:
a) 1 only
Q.10) Consider the following statements: b) 2 only
1. Purana Qila is one of the oldest c) Both 1 and 2
forts belonging to the Gupta era. d) Neither 1 nor 2
.
IASBABA’S AIPTS 2024 – TEST 4 (CURRENT AFFAIRS – JUNE 2023)

Q.13) Consider the following statements Choose the correct code:


about the Agni-1 Missile: a) 1 and 2
1. It is a single-stage, solid-fuel b) 2 and 3
missile. c) 1 and 3
2. It is a road mobile capable of d) 1, 2 and 3
carrying a nuclear warhead.
3. It has a range of 7000–9000 km. Q.16) Consider the following statements
about the KAVACH:
Choose the correct code:
1. It is the world’s cheapest automatic
a) 1 and 2 train collision protection system.
b) 2 and 3 2. It has been developed by Russian
c) 1 and 3 Railways.
d) 1, 2 and 3 3. It is a Safety Integrity Level 4 (SIL-
4) certified technology.
Q.14) Consider the following:
Dam River Choose the correct code:

1. Srisailam Dam Godavari River a) 1 and 2


b) 2 and 3
2. Polavaram Krishna River
c) 1 and 3
Dam
d) 1, 2 and 3
3. Mettur Dam Kaveri River

4. Idukki Arch Periyar River Q.17) Consider the following statements


Dam about Pet Coke:
How many given pairs are correctly
1. It is one of the industrial by-
matched?
products produced during oil
a) One pair
refining.
b) Two pairs
2. China is the world’s biggest
c) Three pairs
consumer of pet coke.
d) Four pairs
3. It is cheaper than coal and a much
more potent pollutant than coal.
Q.15) Consider the following statements
about the Central Electricity Authority: Choose the correct code:
1. It is a statutory
organization constituted under the a) 1 and 2
Electricity (Supply) Act, of 1948. b) 2 and 3
2. It advises the government on c) 1 and 3
matters relating to the National d) 1, 2 and 3
Electricity Policy (NEP).
3. It is the designated authority for
cross border trade of electricity.
.
IASBABA’S AIPTS 2024 – TEST 4 (CURRENT AFFAIRS – JUNE 2023)

Q.18) The Helmand River Treaty is signed Q.21) Consider the following statements
between? regarding the Gulf of Mannar:
1. It is India’s first Marine Biosphere
a) Iraq and Iran Reserve.
b) Afghanistan and Iran 2. It is the first Marine Biosphere
c) Uzbekistan and Iran Reserve in South East Asia.
d) Kazakhstan and Afghanistan Choose the correct code:
a) 1 only
Q.19) Consider the following statements: b) 2 only
c) Both 1 and 2
1. Mosses are the only plants that
d) Neither 1 nor 2
have a multicellular rhizoid.
2. Mosses have a well-developed
Q.22) Consider the following statements
vascular system.
about Necrophilia:
3. Mosses help to soak up rainfall and
1. It is a psychosexual disorder.
maintain moisture in the soil
2. It is a punishable offence under the
below.
Indian Penal Code.
How many of the above statements are Choose the correct code:
correct? a) 1 only
b) 2 only
a) Only one c) Both 1 and 2
b) Only two d) Neither 1 nor 2
c) All three
d) None Q.23) Consider the following statements
about Salt Caverns:
Q.20) Consider the following statements 1. They have very high oil absorbency.
about the National Statistical Office: 2. They can be located closer to the
surface than rock caverns.
1. It works under the Ministry of
3. They are much faster and cheaper
Finance.
than excavating rock caverns.
2. It compiles and releases Consumer
How many of the above statements are
Price Index (CPI).
correct?
3. It compiles and releases the Index
a) Only one
of Industrial Production (IIP).
b) Only two
Choose the correct code:
c) All three
a) 2 and 3
d) None
b) 1 only
c) 1 and 2
Q.24) Consider the following statements:
d) 3 only
1. New Development Bank originated
during the Sixth BRICS Summit
in Fortaleza, Brazil.
.
IASBABA’S AIPTS 2024 – TEST 4 (CURRENT AFFAIRS – JUNE 2023)

2. India is a founding member of Q.27) Consider the following statements


BRICS. about the First Loss Default Guarantee
3. The first BRICS Summit took place (FLDG) System:
in 2009 in the Russian Federation. 1. It is a lending model between a
How many of the above statements are fintech and a regulated entity.
correct? 2. It helps expand the customer base
a) Only one of traditional lenders.
b) Only two 3. It is a safety-net arrangement only
c) All three among non-banking finance
d) None companies (NBFC).
Choose the correct code:
Q.25) Consider the following statements a) 1 and 2
regarding Green Hydrogen: b) 2 and 3
1. It is a clean-burning molecule, c) 1 and 3
which can decarbonise a range of d) 1, 2 and 3
sectors including iron and steel.
2. It can be used in fuel cells to Q.28) Consider the following statements
generate electricity, or power and about the World Food Programme (WFP):
heat. 1. It is founded by the Food and
Choose the correct code: Agriculture Organization
a) 1 only (FAO) and United Nations General
b) 2 only Assembly (UNGA).
c) Both 1 and 2 2. It is headquartered in Rome, Italy.
d) Neither 1 nor 2 3. It is funded entirely by voluntary
donations.
Q.26) Consider the following statements How many of the above statements are
regarding French Open: correct?
1. It is a major Tennis tournament. a) Only one
2. It is one of the four Grand Slams in b) Only two
Tennis. c) All three
3. It is also known as Roland-Garros. d) None
Choose the correct code:
a) 1 only Q.29) Consider the following statements:
b) 2 and 3 1. The sudden rise of life forms.
c) 1 and 3 2. Earth suffered below-zero
d) 1, 2 and 3 temperatures.
3. Earth was not tectonically active.
Which of the following events took place
during the Proterozoic Eon?
a) 1 and 2
b) 2 and 3
.
.
.
.
IASBABA’S AIPTS 2024 – TEST 4 (CURRENT AFFAIRS – JUNE 2023)

Q.47) Consider the following statements Q.50) Consider the following statements
about Van Dhan Vikas Kendras: about the Agra Fort:
1. They are set up under the Pradhan 1. It has been occupied by Rajputs,
Mantri Van Dhan Yojana (PMVDY). Mughals, Jats, and Marathas.
2. They aim to promote Minor Forest 2. It is a UNESCO World Heritage site.
Products (MFPs)-centric livelihood. 3. It comprises the Jahangir Palace
Choose the correct code: and the Khas Mahal.
a) 1 only How many of the above statements are
b) 2 only correct?
c) Both 1 and 2 a) Only one
d) Neither 1 nor 2 b) Only two
c) All three
Q.48) Consider the following statements d) None
about the District Legal Services Authority:
1. It is a statutory body formed
under the Legal Services Q.51) Consider the following statements
Authorities Act, of 1987. regarding E20 fuel:
2. The State Government in 1. It is a blend of 80% ethanol and 20%
consultation with the Chief Justice petrol.
of the High Court constitutes it. 2. It reduces vehicular tailpipe
3. District Judge is appointed as its emissions.
chairman. 3. It increases the import bill for crude
Choose the correct code: oil.
a) 1 and 2 How many of the above statements are
b) 2 and 3 correct?
c) 1 and 3 a) Only one
d) 1, 2 and 3 b) Only two
c) All three
Q.49) Consider the following statements d) None
about the National Institutional Ranking
Framework (NIRF): Q.52) Cry2Ai is mentioned in the context
1. It is a methodology to rank of?
institutions of higher education in a) Cybersecurity
India. b) Artificial Intelligence
2. It is released by the NITI Aayog. c) Transgenic Crop
Choose the correct code: d) None
a) 1 only
b) 2 only
c) Both 1 and 2
d) Neither 1 nor 2
.
IASBABA’S AIPTS 2024 – TEST 4 (CURRENT AFFAIRS – JUNE 2023)

Q.53) Consider the following statements Q.56) Consider the following statements
regarding Sovereign Gold Bond (SGB) regarding the consent principle for the
Scheme: Central Bureau of Investigation (CBI):
1. SGBs are issued by the Reserve 1. It is the consent of the state
Bank of India on behalf of the government given to the CBI that
Government of India. can be either case-specific or
2. Its objective is to mobilize the gold general.
held by households and institutions 2. No states have withdrawn the
in the country. general consent given to the CBI till
3. The tenure of the SGB will be for a today since 1946.
period of five years with an option Choose the correct code:
of premature redemption after a) 1 only
5th year. b) 2 only
Choose the correct code: c) Both 1 and 2
a) 1 and 3 d) Neither 1 nor 2
b) 1 and 2
c) 2 and 3 Q.57) Consider the following statements
d) 1, 2 and 3 about Cargo Release Time:
1. It is the time taken from the arrival
Q.54) Consider the following statements: of the cargo at the customs
1. The first Global Wind Day was station to its out-of-charge.
celebrated in 2023 by the European 2. It is a key indicator of trade
Wind Energy Association (EWEA). efficiency and ease of doing
2. The theme of Global Wind Day business.
2023 was “Pawan - Urja: Powering 3. It is a performance measurement
the Future of India”. tool recommended by the World
Choose the correct code: Trade Organisation.
a) 1 only How many of the above statements are
b) 2 only correct?
c) Both 1 and 2 a) Only one
d) Neither 1 nor 2 b) Only two
c) All three
Q.55) Recently in the news, Manus Island is d) None
located in?
a) Australia Q.58) Which of the following released
b) Indonesia National Time Release Study (NTRS) 2023
c) Sri Lanka report?
d) Papua New Guinea a) World Trade Organisation
b) Central Board of Direct Taxes
(CBDT)
c) World Bank
.
IASBABA’S AIPTS 2024 – TEST 4 (CURRENT AFFAIRS – JUNE 2023)

d) Central Board of Indirect Taxes and Q.61) Consider the following statements
Customs (CBIC) about the National Cooperative Union of
India (NCUI):
Q.59) Consider the following statements 1. It is the apex body representing all
regarding Oslo Forum: sectors of the Indian co-operative
1. It is a series of retreats for movement.
international conflict mediators 2. Its membership is not open to
and high-level decision-makers. multi-state co-operative societies.
2. It is co-hosted by the External 3. It is organising Indian Cooperative
Affairs Ministries of India and Congress (ICC) with the theme
Afghanistan. of Amrit Kaal.
3. All discussions are confidential and How many of the above statements are
take place under the Chatham correct?
House Rule. a) Only one
Choose the correct code: b) Only two
a) 1 and 2 c) All three
b) 2 and 3 d) None
c) 1 and 3
d) 1, 2 and 3 Q.62) Consider the following statements
about the National Research Foundation
Q.60) Consider the following statements (NRF):
regarding Saksham Anganwadi and Poshan 1. It will provide fellowships for post-
2.0: doctoral research.
1. It is a centrally sponsored scheme 2. It will be administratively housed in
implemented by states and union the Department of Science and
territories. Technology (DST).
2. It seeks to address the challenges of 3. The Prime Minister will be the ex-
malnutrition in children, officio president of the NRF.
adolescent girls, pregnant women, 4. It will forge collaborations among
and lactating mothers. the industry, academia,
3. It is being implemented by the government departments, and
Ministry of Health and Family research institutions.
Welfare. Choose the correct code:
Choose the correct code: a) 1, 3 and 4
a) 1 and 2 b) 3 and 4
b) 2 and 3 c) 2 and 3
c) 1 and 3 d) 1, 2, 3 and 4
d) 1, 2 and 3
.
IASBABA’S AIPTS 2024 – TEST 4 (CURRENT AFFAIRS – JUNE 2023)

Q.63) Consider the following statements a) 1 only


about Padma Awards: b) 2 only
1. It was instituted in 1954 to c) Both 1 and 2
recognize achievements in all fields d) Neither 1 nor 2
of activities where an element of
public service is involved. Q.66) Consider the following statements:
2. It is announced every year on the Statement I:
occasion of Independence Day and Cobalt, gallium, graphite, and lithium are
is presented by the President of critical minerals in India.
India. Statement II:
3. Government servants including These are elements that are crucial to
those working with PSUs, except modern-day technologies and are at risk of
doctors and scientists, are not supply chain disruptions.
eligible for these Awards. Which one of the following is correct in
Choose the correct code: respect of the above statements?
a) 1 and 2 a) Both Statement-I and Statement-II
b) 2 and 3 are correct and Statement-II is the
c) 1 and 3 correct explanation for Statement-I
d) 1, 2 and 3 b) Both Statement-I and Statement-II
are correct and Statement-II is not
Q.64) Consider the following statements the correct explanation' for
about the Champions Portal: Statement-I
1. It is a single-window grievance c) Statement I is correct but
redressal portal for Micro, Small & Statement II is incorrect
Medium Enterprises (MSME). d) Statement I is incorrect Statement
2. It was launched by the Ministry of II is correct
Commerce and Industry.
Choose the correct code: Q.67) Consider the following statements:
a) 1 only Statement I:
b) 2 only National Statistics Day is celebrated on
c) Both 1 and 2 29th June every year.
d) Neither 1 nor 2 Statement II:
It is celebrated to commemorate the birth
Q.65) Consider the following statements anniversary of Aryabhatta.
about Kharchi Puja: Which one of the following is correct in
1. It is one of the main festivals of respect of the above statements?
Bihar. a) Both Statement-I and Statement-II
2. It is performed during the months are correct and Statement-II is the
of July-August on the eighth day of correct explanation for Statement-I
the new moon. b) Both Statement-I and Statement-II
Choose the correct code: are correct and Statement-II is not
.
IASBABA’S AIPTS 2024 – TEST 4 (CURRENT AFFAIRS – JUNE 2023)

the correct explanation' for Q.70) Consider the following statements


Statement-I about Just Energy Transition Partnership
c) Statement I is correct but (JET-P) Deal:
Statement II is incorrect 1. It is an initiative to accelerate the
d) Statement I is incorrect Statement phasing out of coal and reduce
II is correct emissions.
2. It was launched at the 27th UN
Q.68) Consider the following statements Climate Change Conference of the
about the Central Board for Film Parties (COP 27).
Certification (CBFC): Choose the correct code:
1. It is a statutory body under a) 1 only
the Cinematograph Act 1952. b) 2 only
2. It works under the Ministry of c) Both 1 and 2
Information and Broadcasting. d) Neither 1 nor 2
3. Films can be publicly exhibited in
India only after they are certified by Q.71) Consider the following statements
the CBFC. about Provident Fund (PF):
Choose the correct code: 1. It is a fixed amount of money that
a) 1 and 2 is contributed by employees from
b) 2 and 3 their salary until retirement.
c) 1 and 3 2. It provides good investment
d) 1, 2 and 3 opportunities to individuals upon
retirement.
Q.69) Consider the following statements 3. It is regulated by the Ministry of
about Green Credit Programme (GCP): Finance.
1. It will be administered by the Indian How many of the above statements are
Council of Forestry Research and correct?
Education (ICFRE). a) Only one
2. Its objective is to create a market- b) Only two
based mechanism for undertaking c) All three
environment-friendly activities. d) None
Choose the correct code:
a) 1 only Q.72) Consider the following statements
b) 2 only about the United Nations Office for Outer
c) Both 1 and 2 Space Affairs (UNOOSA):
d) Neither 1 nor 2 1. It helps countries use space data
and technologies to prevent and
manage disasters.
2. It is headquartered in New York.
3. The Office of the Director (OD)
oversees and coordinates the
.
IASBABA’S AIPTS 2024 – TEST 4 (CURRENT AFFAIRS – JUNE 2023)

strategic direction and operational 3. Red Take Action


priorities of UNOOSA.
Choose the correct code: 4. Green Be Aware
a) 1 and 2 How many given pairs are correctly
b) 2 and 3 matched?
c) 1 and 3 a) One pair
d) 1, 2 and 3 b) Two pairs
c) Three pairs
Q.73) It is the oldest and largest national d) Four pairs
park in Maharashtra. It is Maharastra’s
second tiger reserve with tropical dry Q.76) Consider the following statements
deciduous forest: about Tenzing Norgay National Adventure
The above statements refer to which of the Award-TANNA:
following Tiger Reserve? 1. It is named after one of the first
a) Melghat Tiger Reserve individuals to reach the summit of
b) Tadoba-Andhari Tiger Reserve Mount Everest.
c) Sahyadri Tiger Reserve 2. It is awarded by the Ministry of
d) Pench Tiger Reserve Youth Affairs and Sports.
3. Its objective is to recognize
Q.74) Consider the following statements the achievements of the person in
about Greedflation: the field of adventure.
1. It is the phenomenon How many of the above statements are
where corporate greed becomes a correct?
significant factor in fueling a) Only one
inflation. b) Only two
2. The Indian corporate sector has not c) All three
been affected by it. d) None
Choose the correct code:
a) 1 only Q.77) Consider the following statements
b) 2 only about the Order of the Nile:
c) Both 1 and 2 1. It is South Africa’s highest state
d) Neither 1 nor 2 honour instituted in 1915.
2. It is a pure gold collar consisting
Q.75) Consider the following regarding the of three-square gold
alerts issued by the Indian Meteorological units comprising pharaonic
Department: symbols.
Colour Alert 3. It is conferred upon Heads of state,
Crown Princes, and Vice-
1. Yellow All is Well
Presidents.
2. Orange Be Prepared Choose the correct code:
a) 1 and 2
.
IASBABA’S AIPTS 2024 – TEST 4 (CURRENT AFFAIRS – JUNE 2023)

b) 2 and 3 2. According to NDRF Report, 70% of


c) 1 and 3 the land mass in India is prone to
d) 1, 2 and 3 floods.
Choose the correct code:
Q.78) Consider the following statements a) 1 only
about the Global Environment Facility b) 2 only
(GEF): c) Both 1 and 2
1. It was initially set up as a fund d) Neither 1 nor 2
under World Bank in 1991.
2. It serves as a financial mechanism Q.81) Consider the following statements
for Convention on Biological about Smart City Mission:
Diversity (CBD). 1. It is a joint effort of the Ministry of
3. India is not a founder member of Housing and Urban Affairs
GEF. (MoHUA), and all state and union
Choose the correct code: territories (UT) governments.
a) 1 and 2 2. Its fundamental principles include
b) 2 and 3 the ‘community at the core’ and
c) 1 and 3 ‘more from less’.
d) 1, 2 and 3 3. Its implementation at the city level
will be done by a Special Purpose
Q.79) Consider the following statements Vehicle (SPV).
about Rani Durgavati: How many of the above statements are
1. She was born in 1524 in the Chola correct?
dynasty. a) Only one
2. Madan Mahal Fort in Jabalpur is b) Only two
associated with Rani Durgavati. c) All three
3. The University of Jabalpur was d) None
renamed as Rani Durgavati
Vishwavidyalaya. Q.82) Consider the following statements
Choose the correct code: about the Mahadayi River:
a) 1 and 2 1. It originates in the Western
b) 2 and 3 Ghats from the Bhimgad Wildlife
c) 1 and 3 Sanctuary in Karnataka.
d) 1, 2 and 3 2. Its left-bank tributaries are the
Tambaraparani River, Bainganga
Q.80) Consider the following statements River, and Wardha River.
about Flash Floods: 3. The Salim Ali Bird Sanctuary is
1. It is a rapid flooding of geomorphic situated on it.
low-lying areas like lakes and Choose the correct code:
washes. a) 1 and 2
b) 2 and 3
.
IASBABA’S AIPTS 2024 – TEST 4 (CURRENT AFFAIRS – JUNE 2023)

c) 1 and 3 3. It consists of a cash award


d) 1, 2 and 3 of 50000/-, a certificate, and a
medal.
Q.83) Consider the following statements Choose the correct code:
about Kedarnath Temple: a) 1 and 2
1. It is located on the Garhwal b) 2 and 3
Himalayan range near the c) 1 and 3
Mandakini River. d) 1, 2 and 3
2. It forms one of the four sites of the
Chota Char Dham Pilgrimage Q.86) Which part of the Constitution of
circuit. India contains ‘Emergency Provisions’?
3. It is one of the 12 Jyotirlingas of a) Part XVIII
Lord Shiva in India. b) Part XVII
Choose the correct code: c) Part XIIV
a) 1 and 2 d) Part XIV
b) 2 and 3
c) 1 and 3 Q.87) Consider the following statements
d) 1, 2 and 3 about the Artemis Accord:
1. It is a non-binding set of principles
Q.84) Consider the following statements: designed to guide military space
1. Debrigarh Wildlife Sanctuary is exploration.
located in Assam. 2. Its founding members include
2. Hirakud Dam is on the Mahanadi Australia, Canada, Italy, and
River. Luxembourg.
3. Simlipal is the largest national park Choose the correct code:
in Odisha. a) 1 only
Choose the correct code: b) 2 only
a) 2 only c) Both 1 and 2
b) 1 and 2 d) Neither 1 nor 2
c) 2 and 3
d) 1, 2 and 3 Q.88) Consider the following statements
about the Food Corporation of India (FCI):
Q.85) Consider the following statements 1. It is a statutory body under the
about Florence Nightingale Awards: Food Corporation Act of 1964.
1. It is the highest national distinction 2. It works under the Ministry of
a doctor can achieve for Consumer Affairs, Food, and Public
exceptional professionalism. Distribution.
2. It is given by the Ministry of Health Choose the correct code:
and Family Welfare. a) 1 only
b) 2 only
c) Both 1 and 2
.
IASBABA’S AIPTS 2024 – TEST 4 (CURRENT AFFAIRS – JUNE 2023)

d) Neither 1 nor 2 Q.91) Which of the following are the


functions of the Bureau of Indian
Q.89) Consider the following statements Standards (BIS)?
about the composition of the National 1. Standards formulation
Human Rights Commission (NHRC): 2. Product Certification
1. It is a multi-member 3. Hallmarking
body consisting of a chairperson, 4. Laboratory services
five full-time members, and seven Choose the correct code:
deemed members. a) 2 and 3 only
2. They serve for five years or until b) 1, 3 and 4
they attain the age of 70 years, c) 3 and 4 only
whichever is earlier. d) 1, 2, 3 and 4
3. They can be removed only on the
charges of proven misbehavior or Q.92) Consider the following statements
incapacity. about the functions of the Directorate of
Choose the correct code: Civil Aviation (DGCA):
a) 1 and 2 1. It issues licenses to pilots, aircraft
b) 2 and 3 maintenance engineers, and flight
c) 1 and 3 engineers.
d) 1, 2 and 3 2. It formulates standards of
airworthiness for civil
Q.90) Consider the following statements aircraft registered in India.
about the ‘United Nations Educational, How many of the above statements are
Scientific and Cultural Organisation’ correct?
(UNESCO): a) 1 only
b) 2 only
1. India is a founding member of
c) Both 1 and 2
UNESCO.
d) Neither 1 nor 2
2. India got re-elected to UNESCO
executive board for 2021-25 term.
Q.93) The Global Gender Gap Report 2023
3. United States of America is not a
is released by the?
member of UNCESO.
a) World Economic Forum
How many of the above statements are b) United Nations
correct? c) World Bank
a) Only one d) World Health Organisation
b) Only two
c) All three Q.94) The Canary Islands are located in?
d) None a) Italy
b) France
c) Germany
d) Spain
.
IASBABA’S AIPTS 2024 – TEST 4 (CURRENT AFFAIRS – JUNE 2023)

Q.95) Which of the following are Q.98) Which constitutional amendment


Mesolithic Sites? conferred the Union Territory of Delhi with
1. Ganeshwar in Rajasthan a special status?
2. Hatkhamba in Karnataka a) 56th Constitutional Amendment
3. Paisra in Bihar Act of 1971
4. Kuchai in Odisha b) 97th Constitutional Amendment
Choose the correct code: Act of 2011
a) 1, 2 and 4 c) 89th Constitutional Amendment
b) 2 and 4 Act of 2001
c) 1, 3 and 4 d) 69th Constitutional Amendment
d) 2 and 3 Act of 1991

Q.96) Consider the following statements Q.99) Consider the following statements
about the characteristics of Mesolithic about the Statue of Unity:
Rock Paintings: 1. It was built in the honour
1. They mainly saw the use of red of Subhash Chandra Bose.
colour. 2. It is the tallest statue in the world.
2. The hunting scenes were 3. It is located in Sadhu Bet Island,
predominant in the period. Gujarat.
3. Animals depicted include How many of the above statements are
elephants, bison, tiger, and boar. correct?
How many of the above statements are a) Only one
correct? b) Only two
a) Only one c) All three
b) Only two d) None
c) All three
d) None Q.100) The Gangotri National Park (GNP) is
located in which of the Indian state?
Q.97) Consider the following statements a) Uttarakhand
about GEMCOVAC-OM: b) Himachal Pradesh
1. It is India’s first mRNA-based c) Ladakh
Omicron-specific booster vaccine. d) Jammu and Kashmir
2. It works by introducing a piece of
DNA that corresponds to a viral
protein.
How many of the above statements are
correct?
a) 1 only
b) 2 only
c) Both 1 and 2
d) Neither 1 nor 2
.
IASBABA’S AIPTS 2024 – TEST 4 (CURRENT AFFAIRS – JUNE 2023) SOLUTIONS

Q.1) Solution (a)


Statement Analysis:

• Delimitation means the act or process of fixing limits or boundaries of territorial


constituencies in a country or a province having a legislative body.
• The Delimitation Commission is appointed by the President of India and works in
collaboration with the Election Commission of India (ECI). Hence statement 1 is
incorrect.
• Delimitation Commissions have been set up four times — 1952, 1963, 1973, and 2002
under the Acts of 1952, 1962, 1972, and 2002.
• Delimitation for LS (Lok Sabha) and LA (Legislative Assembly) is different from that
of Local bodies.
• The Delimitation Commission Act was enacted in 1952.
• The first delimitation exercise was carried out by the President (with the help of the
Election Commission) in 1950-51.

Composition of Delimitation Commission:

• Retired Supreme Court judge


• Chief Election Commissioner
• Respective State Election Commissioners
• The Constitution of India mandates that the allocation of seats in the Lok Sabha should
be based on the population of each state so that the ratio of seats to population is as
close as possible to being equal across all states. It is intended to ensure that each
person's vote carries roughly the same weight, regardless of which state they live in.
Hence statement 2 is correct.
• The 87th Amendment Act of 2003 provided for the delimitation of constituencies
based on the 2001 census and not the 1991 census.

Constitutional Provisions:

• Under Article 82, the Parliament enacts a Delimitation Act after every Census.
• Under Article 170, States also get divided into territorial constituencies as per
Delimitation Act after every Census. Hence statement 3 is incorrect.

Source: https://www.thehindu.com/news/national/telangana/delimitation-based-on-
population-is-irrational-injustice-to-southern-states-ktr/article66911309.ece

Q.2) Solution (a)


Statement Analysis:

• Evergreening loans, a form of zombie lending, is a practice of extending new or


additional loans to a borrower who is unable to repay the existing
.
IASBABA’S AIPTS 2024 – TEST 4 (CURRENT AFFAIRS – JUNE 2023) SOLUTIONS

loans, thereby concealing the true status of the non-performing assets (NPAs) or bad
loans.

Approaches Utilised for Evergreening Loans:


✓ Selling and buying back loans or debt instruments between two lenders to avoid
classifying them as NPAs.
✓ Persuading good borrowers to enter into structured deals with stressed borrowers to
hide their default.
✓ Using internal or office accounts to adjust the repayment obligations of borrowers.
✓ Renewing or disbursing new loans to stressed borrowers or related entities closer to
the repayment date of earlier loans. Hence statement 1 is correct.
• Evergreening loans can create a false impression of the asset quality and profitability
of banks and delay the recognition and resolution of stressed assets.
• It can also undermine credit discipline and moral hazard among borrowers, and erode
the trust and confidence of depositors, investors, and regulators. Hence statement 2
is correct.

Note: NPA refers to a classification for loans or advances that are in default or are in arrears
on scheduled payments of principal or interest. Banks are required to classify non-performing
assets further into the following three categories based on the period for which the asset has
remained non-performing and the realizability of the dues:

• Sub-standard Assets: A substandard asset is an asset classified as an NPA for a period


less than or equal to 12 months
• Doubtful Assets: A doubtful asset is an asset that has been nonperforming for a period
exceeding 12 months.
• Loss Assets: Assets that are uncollectible and where there is little, or no hope of
recovery and that needs to be fully written off.
Source: https://indianexpress.com/article/explained/explained-economics/banks-
corporates-window-dressing-loans-explained-rbi-8636026/

Q.3) Solution (c)


Statement Analysis:

• Graphene is a one-atom-thick layer of carbon atoms arranged in a hexagonal lattice.


As the building block of graphite, graphene possesses remarkable characteristics that
make it a potential game-changer in various industries.
• Graphene’s capability to absorb and dissipate electromagnetic waves positions it as a
valuable component for stealth coatings and materials that reduce radar signatures
and electromagnetic interference. Hence statement 1 is correct.

Its properties:
✓ It surpasses copper in electrical conductivity, being 200 times stronger than steel yet
six times lighter.
.
IASBABA’S AIPTS 2024 – TEST 4 (CURRENT AFFAIRS – JUNE 2023) SOLUTIONS

✓ With exceptional transparency of absorbing only 2% of light, it also exhibits


impermeability to gases, including hydrogen and helium.
✓ The high surface-area-to-volume ratio of graphene, stemming from its thinness,
makes it a promising candidate for energy storage.
• Graphene's sensitivity to environmental changes makes it an excellent candidate for
• sensing chemical and biological agents, explosives, radiation, and other hazardous
substances.
• It also holds promise in numerous applications such as efficient and precise sensors,
faster and more efficient electronics, efficient solar panels, accelerated DNA
sequencing, and drug delivery systems. Hence statement 2 is correct.
• It has been proven that even a minuscule amount of graphene added to plastics,
metals, or other materials can significantly enhance their strength.
• These graphene composites find applications in aerospace, automotive, sports
equipment, construction, high-performance batteries, conductive inks, water
purification, desalination, and even in the production of graphene-based masks during
the COVID-19 pandemic. Hence statement 3 is correct.
Source: https://www.thehindu.com/opinion/op-ed/is-india-missing-the-graphene-
bus/article66915310.ece

Q.4) Solution (b)


Statement Analysis:

• Sustainable Aviation Fuel (SAF), also referred to as Bio-Jet Fuel, is created


using domestically developed methods using cooking oil and oil-rich seeds from
plants.
• The CSIR (Council of Scientific and Industrial Research)-IIP (Indian Institute of
Petroleum) has created fuel using different materials, such as non-edible and edible
oils, as well as used cooking oil.
• They used various sources, including palm stearin, sapium oil, palm fatty
acid distillates, algae oil, karanja, and jatropha. Hence statement 1 is correct.

• SAF has the greatest potential to reduce CO2 emissions from International Aviation.
• Depending on the feedstock and technologies used to produce it, SAF can reduce life
cycle GHG emissions dramatically compared to conventional jet fuel.
• Some emerging SAF pathways even have a net-negative GHG footprint. Hence
statement 2 is correct.

• SAF can be blended at up to 50% with traditional jet fuel and all quality tests are
• completed as per traditional jet fuel.
• SAF is currently more costly than traditional fossil jet fuel. It has the potential to
provide a lifecycle carbon reduction of up to 80% compared to the traditional jet fuel
it replaces. Hence statement 3 is incorrect.
Source: https://www.thehindu.com/news/national/countries-must-be-allowed-to-prepare-
own-plans-for-saf-india/article66916574.ece

Q.5) Solution (b)


.
IASBABA’S AIPTS 2024 – TEST 4 (CURRENT AFFAIRS – JUNE 2023) SOLUTIONS

Statement Analysis:

• The City Investments to Innovate, Integrate and Sustain (CITIIS) 2.0 aims to promote a
circular economy.
• The programme envisages to support competitively selected projects promoting a
circular economy with a focus on - Integrated waste management at the city level,
Climate-oriented reform actions at the state level
• Institutional strengthening and knowledge dissemination at the national level. Hence
statement 1 is correct.

• CITIIS, a sub-component of Smart Cities Mission, is a joint program of the Ministry of


Housing and Urban Affairs, Agence Francaise de Development (AFD), European Union
(EU), and National Institute of Urban Affairs (NIUA). CITIIS 2.0 will supplement the
climate actions of the Government of India through its ongoing National programs
(National Mission on Sustainable Habitat, AMRUT 2.0,
• Swachh Bharat Mission 2.0 and Smart Cities Mission), as well as contributing positively
to India’s Intended Nationally Determined Contributions (INDCs) and Conference of
the Parties' (COP26) commitments. Hence statement 2 is correct.

Note: CITIIS 1.0 was launched in 2018 and assisted 12 cities across India in sustainable urban
infrastructure projects. They were Agartala, Amaravati, Amritsar, Bhubaneshwar, Chennai,
Dehradun, Hubbali-Dharwad, Kochi, Puducherry, Surat, Ujjain, and Visakhapatnam.

Source: https://www.thehindu.com/news/national/cabinet-approves-citiis-20-for-
integrated-waste-management-and-climate-oriented-reform/article66916174.ece

Q.6) Solution (a)


Statement Analysis:

• A new report, ‘The Global Slavery Index 2023’, by the Walk Free Foundation, highlights
the increasing prevalence of modern slavery worldwide. The report identifies climate
change, armed conflict, weak governance, and health emergencies like the Covid-19
pandemic as key factors that have contributed to the rise in modern slavery.
• It ranks 160 countries based on their estimated prevalence of modern slavery per
1,000 people.
• The countries with the highest prevalence are North Korea (104.6), Eritrea (90.3), and
Mauritania (32.0), where modern slavery is widespread and often state-sponsored.
• Asia and the Pacific have the largest number of people in modern slavery. Hence
statement 1 is correct.

• The countries with the lowest prevalence are Switzerland (0.5), Norway (0.5), and
Germany (0.6), where strong governance and effective responses to modern slavery
are evident. Hence statement 2 is incorrect.
• According to the Global Slavery Index 2023, an estimated 50 million people were living
in modern slavery on any given day in 2021, an increase of 10 million people since
2016.
.
IASBABA’S AIPTS 2024 – TEST 4 (CURRENT AFFAIRS – JUNE 2023) SOLUTIONS

• This means that one in every 160 people in the world is a victim of modern slavery.
Hence statement 3 is incorrect.
Note: Modern slavery encompasses various forms of exploitation, including forced labour,
forced marriage, debt bondage, commercial sexual exploitation, human trafficking, slavery-
like practices, and the sale and exploitation of children.

Source: https://www.thehindu.com/news/national/g20-countries-including-india-are-
fuelling-modern-slavery-says-new-
report/article66895654.ece#:~:text=The%20Global%20Slavery%20Index%202023,people%2
0living%20in%20'modern%20slavery'&text=50%20million%20people%20are%20living,years
%2C%20says%20a%20new%20report

Q.7) Solution (c)


Statement Analysis:

• The World Health Assembly (WHA) is the WHO’s decision-making body attended by
delegations from all of WHO’s member states.
Functions of WHA:
✓ Deciding on Organization’s policies.
✓ Appointment of the Director-General of WHO.
✓ Administration of financial policies.
✓ Review and approval of the proposed programme budget.
Hence statement 1 is correct.
• Recently, the 76th annual World Health Assembly was held at World Health
Organization (WHO) headquarters (HQ), Geneva, Switzerland, from 21st to
30th May 2023.
• The theme for 2023 is "WHO at 75: Saving lives, driving health for all". Hence
statement 2 is incorrect.
Source: https://www.downtoearth.org.in/news/health/who-member-states-agree-on-6-83-
billion-funding-for-2-years-most-ambitious-yet-89515

Q.8) Solution (c)


Statement Analysis:

• Article 299 of the Constitution deals with the manner and form of contracts made by
or on behalf of the government of India or any state government.
• It outlines the manner in which contracts made in the exercise of the executive power
of the Union or a State shall be expressed and executed.
• It aims to establish a specific procedure to safeguard public funds and prevent
unauthorized or illegitimate contracts.
• According to Article 299(1), contracts must be expressed in writing and executed by a
person duly authorized by the President or the Governor on their behalf. Hence
statement 1 is correct.

• The government had been entering into contracts even in the pre-independence era.
.
IASBABA’S AIPTS 2024 – TEST 4 (CURRENT AFFAIRS – JUNE 2023) SOLUTIONS

• The Crown Proceedings Act of 1947 played a role in shaping Article 299.
• The Crown Proceedings Act specified that the Crown could not be sued in court for a
contract it entered. Hence statement 2 is correct.
The judgements related to Article 299 -

• Mrs. Aliakutty Paul vs The State of Kerala and Ors (1995):


o A tender of the contract for the construction of a bridge was accepted by the
Executive Engineer, but he did not sign it in the name of the Governor, it
cannot be said that there is a valid contract in conformity with Article 299 of
the Constitution.
o The decision explains the rationale and scope of Article 299 of the Constitution
and emphasizes that its provisions are enacted for safeguarding the
government against unauthorized contracts.
• State of Bihar v Majeed (1954):
o SC ruled that a Government Contract has to comply with the provisions of
Article 299 in addition to the requirements of the Indian Contract Act, such as
offer, acceptance, and consideration.
• The contractual liability of the Central or State Government is the same as that of any
individual under the ordinary law of contract, subject to the formalities prescribed by
Article 299. Hence statement 3 is correct.
Source: https://indianexpress.com/article/explained/explained-law/govt-immunity-
contract-president-name-sc-8636329/

Q.9) Solution (a)


Statement Analysis:

• Direct Seeded Rice (DSR), also known as the 'broadcasting seed technique,' is a water-
saving method of sowing paddy.
• With the use of drum seeders, only two laborers are required to sow seeds on one
acre using the DSR method, compared to 25-30 laborers needed in traditional
methods. This significantly reduces labor costs and eases the burden on farmers.
Hence statement 1 is correct.

• In this method, seeds are directly drilled into the fields, eliminating the need for
nursery preparation and transplantation. Hence it reduces labor cost.
Other benefits:
✓ By eliminating the need for nursery cultivation, farmers save approximately 30 days in
the crop cycle.
✓ This allows them to start the rabi season early and avoid untimely rains during the
harvesting phase.
✓ The direct-seeding method reduces water requirements by around 15% as water
logging occurs only after a month. This is especially beneficial in areas where rainfall
is delayed.
Hence statement 2 is correct.
.
IASBABA’S AIPTS 2024 – TEST 4 (CURRENT AFFAIRS – JUNE 2023) SOLUTIONS

Source: https://www.thehindubusinessline.com/economy/agri-business/direct-seeding-
method-gains-currency-in-indias-paddy-growing-regions/article66908422.ece

Q.10) Solution (b)


Statement Analysis:

• Purana Qila is one of the oldest forts belonging to the Mughal era and the site is
identified as an ancient settlement of Indraprastha (the capital of Pandavas).
• The massive gateway and walls of Purana Qila were built by Humayun in the
16th century and the foundation was laid for the new capital, Dinpanah.
• The work was carried forward by Sher Shah Suri, who displaced Humayun. Hence
statement 1 is incorrect.
• The major attractions inside the fort are the Qila-i-Kuhna mosque of Sher Shah
Suri, Sher Mandal (a tower, which is traditionally associated with the death of
Humayun), a stepwell, and the remains of the extensive rampart, which has three
gates.
• The unique features of Indo-Islamic architecture like horseshoe-shaped arches,
bracketed openings, marble inlay, carving, etc are very prominent in the structure.
• The mosque has an inscription that says, 'As long as there are people on this earth,
may this edifice be frequented, and people will be happy in it.' Hence statement 2 is
correct.

• The recent excavations by the Archaeological Survey of India (ASI) at Purana Qila (Old
Fort) in Delhi have revealed a continuous history spanning over 2,500 years. These
excavations are aimed to establish a complete chronology of the site.
• Artifacts from various historical periods have been discovered, which revealed 9
cultural levels including Pre-Mauryan, Mauryan, Sunga, Kushana, Gupta, Post-Gupta,
Rajput, Sultanate, and Mughal.

The findings of the excavation are:


✓ A well from the Mauryan period was unearthed.
✓ A 900-year-old sculpture of Vaikuntha Vishnu, belonging to the Rajput period, was
discovered.
✓ A terracotta plaque depicting the goddess Gaja Lakshmi, dating back to the Gupta
period, was found.
✓ A well-defined four-room complex from the Sunga-Kushan period, which dates back
approximately 2,300 years was unearthed.
✓ More than 136 coins, 35 seals and sealings, and other copper artifacts were
discovered.
Hence statement 3 is correct.
Source: https://www.thehindu.com/news/national/purana-qila-excavations-reveal-pre-
mauryan-era-settlement/article66912593.ece
.
IASBABA’S AIPTS 2024 – TEST 4 (CURRENT AFFAIRS – JUNE 2023) SOLUTIONS

Q.11) Solution (b)


Statement Analysis:

• Mission Vatsalya is an umbrella scheme for child protection services in the country.

Its Objectives: -
✓ To secure a healthy and happy childhood for each and every child in the country.
✓ To ensure opportunities to enable them to discover their full potential.
✓ To assist States/UTs in delivering the mandate of the Juvenile Justice Act 2015.
✓ To achieve the Sustainable Development (SDG)goals.
✓ It promotes family-based non-institutional care of children in difficult circumstances.
Hence statement 1 is incorrect.
• It is implemented by the Ministry of Women and Child Development.
Under the mission, the Government plans to partner with the private sector as well as
volunteer groups. Hence statement 2 is correct.
Its Components: -
✓ Strengthening service delivery structures
✓ Upscaling institutional care and services
✓ Encouraging non-institutional community-based childcare
✓ Emergency outreach services
✓ Training and capacity building
Hence statement 3 is correct.
.
IASBABA’S AIPTS 2024 – TEST 4 (CURRENT AFFAIRS – JUNE 2023) SOLUTIONS

Source: https://pib.gov.in/PressReleasePage.aspx?PRID=1929348

Q.12) Solution (a)


Statement Analysis:

• The lightweight’ payment system is to ensure uninterrupted digital payments during


emergencies such as natural calamities or war.
• It can be operated from anywhere.
• It requires minimal staff during emergencies.
• It will process critical transactions, such as bulk payments and interbank
payments, during extreme and volatile situations. Hence statement 1 is correct.
• This system will operate independently of existing payment technologies like UPI
(Unified Payment Interface), NEFT (National Electronic Funds Transfer), and RTGS
(Real-Time Gross Settlement (RTGS). Hence statement 2 is incorrect.
• The system operates on minimalistic hardware and software.
• It gets activated only when needed.
Source: https://indianexpress.com/article/explained/explained-sci-tech/what-is-rbi-
planned-lightweight-payments-system-for-emergencies-8637007/

Q.13) Solution (a)


Statement Analysis:

• Agni-1 is a single-stage, solid-fuel missile.


.
IASBABA’S AIPTS 2024 – TEST 4 (CURRENT AFFAIRS – JUNE 2023) SOLUTIONS

• It was first test-fired in 2002.


• It has been successfully test-fired 11 times.
• It is 15 meters long and weighs 12 tonnes.
• It is powered by a solid-fuel rocket motor. Hence statement 1 is correct.
• It is road-mobile, which gives it a high degree of mobility and makes it difficult for an
enemy to target.
• It is capable of carrying a nuclear warhead.
• It is a part of India’s nuclear deterrent.
• The Strategic Forces Command (SFC) of the Indian Army uses it. Hence statement 2 is
correct.
• This missile has the capability of striking targets with a very high degree of precision.
• It has a range of 700–900 km, which gives it the ability to strike targets in Pakistan and
China.
The latest upgrade to the Agni-1 missile includes: -
✓ a new guidance system that makes it more accurate and
✓ a new warhead that increases its destructive power.
Hence statement 3 is incorrect.

• Note: The Strategic Forces Command (SFC) is a tri-Service command that forms part
of India’s Nuclear Command Authority (NCA). It was created on January 4, 2003, by an
executive order of the Cabinet Committee on Security (CCS) headed by then Prime
Minister Atal Bihari Vajpayee. It is responsible for the management and administration
of the country’s tactical and strategic nuclear weapons stockpile. A Commander-in-
Chief of the rank of Air Marshal heads it.
Source: https://www.financialexpress.com/business/defence-successful-training-launch-of-
mrbm-agni-1-know-more-about-the-missile-3111164/

Q.14) Solution (b)


Statement Analysis:

Dam River

1. Srisailam Dam Krishna River

2. Polavaram Dam Godavari River

3. Mettur Dam Kaveri River

4. Idukki Arch Dam Periyar River

Hence option b is correct.


Source: https://www.newindianexpress.com/states/tamil-nadu/2023/jun/02/mettur-dam-
water-release-datenears-tn-farmersbusy-procuring-seeds-and-fertilisers-2581085.html

Q.15) Solution (d)


.
IASBABA’S AIPTS 2024 – TEST 4 (CURRENT AFFAIRS – JUNE 2023) SOLUTIONS

Statement Analysis:

• The Central Electricity Authority (CEA) is a statutory organization constituted


under Section 3 (1) of the repealed Electricity (Supply) Act, of 1948. Hence statement
1 is correct.
• It later continued under Section 70 of the Electricity Act, of 2003.
• Background: It was established as a part-time body in 1951 and made a full-time body
in the year 1975.
• As per Section 70 (3) of the Electricity Act, 2003, the authority shall consist of not more
than 14 members, including its chairperson.
• Not more than eight shall be full-time members appointed by the Central
Government.
• Its chairperson is the Chief Executive of the Authority.

Its functions: -
✓ It advises the government on matters relating to the National Electricity Policy (NEP).
Hence statement 2 is correct.
✓ It formulates short-term and prospective plans for the development of electrical
systems.
✓ It is the designated authority for cross border trade of electricity. Hence statement 3
is correct.
✓ It also prescribes the standards on matters such as the construction of electrical
plants, electric lines and connectivity to the grid, safety, and grid standards, and
installation and operation of meters.
✓ It is also responsible for the concurrence of hydropower development schemes of
central, state, and private sectors for the efficient development of rivers and their
tributaries for power generation.
Source: https://www.thehindu.com/news/national/half-of-indias-electric-capacity-from-
renewables-by-2027-cea/article66920304.ece

Q.16) Solution (c)


Statement Analysis:

• KAVACH is an Automatic Train Protection (ATP) electronic system designed to help the
Indian Railways achieve Zero Accidents.
• Its objective is to achieve safety in train operations across Indian Railways.
• KAVACH is the world’s cheapest automatic train collision protection system. Hence
statement 1 is correct.
• KAVACH has been indigenously developed by the Research Design and Standards
Organisation (RDSO) in collaboration with the Indian industry. Hence statement 2 is
incorrect.
• KAVACH is a Safety Integrity Level 4 (SIL-4) certified technology.
• Safety Integrity Level (SIL) comes from two voluntary standards used by plant
owners/operators to quantify safety performance requirements for hazardous
operations.
.
IASBABA’S AIPTS 2024 – TEST 4 (CURRENT AFFAIRS – JUNE 2023) SOLUTIONS

• There are four SIL Levels (1-4).


• A higher SIL Level means a greater process hazard and a higher level of protection.
Hence statement 3 is correct.

Note: Research Design and Standards Organisation (RDSO) is a research and development
organisation under the Ministry of Railways.
Source: https://www.thehindubusinessline.com/blexplainer/what-is-kavach-could-it-have-
prevented-the-odisha-train-tragedy/article66926860.ece

Q.17) Solution (c)


Statement Analysis:

• Pet Coke is one of the industrial by-products produced during oil refining.
• It is dark solid carbon material.
• It is used as a source of energy and carbon for various industrial applications.
• It is abundantly used in India in several manufacturing industries such as cement,
steel, and textile. Hence statement 1 is correct.
• There are two kinds of pet coke produced during oil refining viz.
✓ Fuel-grade pet coke (80%) and
✓ Calcined pet coke (20%).
• India is the world’s biggest consumer of pet coke.
• Local producers include Indian Oil Corp, Reliance Industries, and Bharat Petroleum
Corp. Hence statement 2 is incorrect.
• Pet Coke is significantly cheaper than coal, has high calorific value, and is easier to
transport and store.
• Pet coke is a much more potent pollutant than coal.
• It contains a whopping 74,000 PPM of sulphur content, which is released into the
atmosphere as an emission, which is much higher than vehicular emissions.
• It is also a source of fine dust, which can get through the filtering process of the human
airway and lodge in the lungs, which can cause serious health problems.
• It releases sulphur, nitrous oxide, mercury, arsenic, chromium, nickel, and hydrogen
chloride, which contribute to global warming. Hence statement 3 is correct.
.
IASBABA’S AIPTS 2024 – TEST 4 (CURRENT AFFAIRS – JUNE 2023) SOLUTIONS

Source: https://www.business-standard.com/economy/news/govt-permits-import-of-pet-
coke-as-raw-material-for-lithium-ion-batteries-123060200960_1.html

Q.18) Solution (b)


Statement Analysis:

• Afghanistan and Iran signed the Helmand River Treaty in 1973. Hence option b is
correct.
• Its objective is to regulate the allocation of river water.
• However, the accord was neither ratified nor fully implemented, causing
disagreements and tensions to persist.
• The Helmand is Afghanistan’s longest river.
• It originates near Kabul (Afghanistan) in the western Hindu Kush Mountain range.
• It flows in a south-westerly direction through desert areas.
• It empties into Lake Hamun, which straddles the Afghanistan-Iran border.

Note: Lake Hamun is the largest freshwater lake in Iran. It has experienced a drastic decline
in water levels, attributed to factors such as drought and the construction of dams. It supports
agricultural activities, livelihoods, and economic sectors in the surrounding areas.
Source: https://indianexpress.com/article/explained/explained-global/what-is-the-iran-
taliban-water-conflict-all-about-8642887/lite/

Q.19) Solution (b)


Statement Analysis:

• Mosses are small, soft, flowerless plants.


• It belongs to Kingdom – Plantae and Division – Bryophyta
• They have Stomata, cells specialized for photosynthetic gas exchange.
• Mosses are the only Plants that have a Multicellular Rhizoid.
• Rhizoid is a root-like subterranean tissue that absorbs water and nutrients from the
soil. Hence statement 1 is correct.
• Mosses are non-vascular plants which means they lack a vascular system i.e. xylem
and phloem. Hence statement 2 is incorrect.
• Mosses can also be extremely useful as Habitat Indicators.
• Mosses are an amazingly resilient and versatile group of Plants.
• They do not have flowers or seeds.
• Mosses function like sponges, using their capillary spaces to hang on to water.
.
IASBABA’S AIPTS 2024 – TEST 4 (CURRENT AFFAIRS – JUNE 2023) SOLUTIONS

• They help to soak up rainfall, maintain moisture in the soil below and keep conditions
around them humid. This enables other plants around them to thrive, such as in
habitats like marshes and woodland. Hence statement 3 is correct.

Source: https://www.downtoearth.org.in/blog/wildlife-biodiversity/an-epic-global-study-of-
moss-reveals-it-is-far-more-vital-to-earth-s-ecosystems-than-we-knew-89678

Q.20) Solution (a)


Statement Analysis:

• The National Sample Survey Office (NSSO) merged with the Central Statistical Office
(CSO) to form the National Statistical Office (NSO) in 2019.
• NSO is under the Ministry of Statistics and Programme Implementation (MoSPI).
Hence statement 1 is incorrect.
• The NSO acts as the nodal agency for the planned development of the statistical
system in the country.
• It lays down and maintains norms and standards in the field of statistics.
• It coordinates the statistical work in respect of the Ministries/Departments of the
Government of India.
• It prepares national accounts as well as publishes annual estimates of national
products, Government and Private final consumption expenditure, Capital Formation,
Savings, etc.
• It compiles and releases Consumer Price Index (CPI). Hence statement 2 is correct.
• It maintains liaison with International Statistical Organizations.
• It compiles and brings out reports as per international/regional commitments.
• It compiles and releases the Index of Industrial Production (IIP) every month. Hence
statement 3 is correct.
• It provides statistical information to assess and evaluate the changes in the growth,
composition, and structure of the organized manufacturing sector.
Source: https://www.thehindu.com/opinion/editorial/riding-the-momentum-the-hindu-
editorial-on-national-statistical-offices-economic-data-and-the-
economy/article66920345.ece

Q.21) Solution (c)


Statement Analysis:

• Gulf of Mannar is an inlet of the Indian Ocean, between south-eastern India and
western Sri Lanka.
.
IASBABA’S AIPTS 2024 – TEST 4 (CURRENT AFFAIRS – JUNE 2023) SOLUTIONS

• It is bounded to the northeast by Rameswaram (island), Adam’s (Rama’s) Bridge (a


chain of shoals), and Mannar Island.
• The gulf receives several rivers, including the Tambraparni (India) and the Aruvi (Sri
Lanka).
• The port of Tuticorin is on the coast.
• It has been renowned for its highly productive pearl banks and religious significance.
• The Government of India designated it as the country’s first Marine Biosphere Reserve
in 1989. Hence statement 1 is correct.
• The Gulf of Mannar National Park has also been identified as the important Bird
Area by BNHS-Birdlife International because of its rich avian fauna.
• The Gulf of Mannar Biosphere Reserve has been identified as an Important Marine
Mammals Area of the World by IUCN due to its dugong population and other marine
mammals' presence.
• The Gulf of Mannar became the first Marine Biosphere Reserve in South East Asia.
Hence statement 2 is correct.
Source: https://newsonair.gov.in/News?title=Gulf-of-Mannar%3a-A-Marine-Biodiversity-
Marvel&id=462047

Q.22) Solution (a)


Statement Analysis:

• Necrophilia is a psychosexual disorder, classified under the DSM-IV, among a group of


disorders, called “paraphilias”.
• Paraphilias include disorders like paedophilia, exhibitionism, and sexual masochism.
• Necrophilia could be the result of rage, experimentation, or lust rather than sexual
necessity or habit. Hence statement 1 is correct.
• The provisions of Sections 375 and 377 of the Indian Penal Code make it clear that
the dead body cannot be called a human or person. Thereby, these provisions of the
IPC would not apply.
• Recently, the Karnataka High Court clarified that sexual intercourse on a dead body is
nothing but necrophilia.
• Hence, it is not a punishable offence under Section 376 (punishment for rape) of the
Indian Penal Code. Hence statement 2 is incorrect.
Source: https://indianexpress.com/article/explained/explained-law/what-is-necrophilia-
and-is-it-an-offence-in-india-8644222/

Q.23) Solution (b)


Statement Analysis:

• Salt caverns are created by dissolving salt deposits with water, and then pumping out
the brine (salt water) to create a hollow space.
• Salt caverns are naturally well-sealed.
• The salt lining acts as a natural barrier against liquid and gas migration, preventing oil
from escaping or contaminating groundwater.
• They have low oil absorbency, which prevents leakage and contamination of the
stored oil. Hence statement 1 is incorrect.
.
IASBABA’S AIPTS 2024 – TEST 4 (CURRENT AFFAIRS – JUNE 2023) SOLUTIONS

• Salt caverns can also be located closer to the surface than rock caverns, which reduces
the drilling costs and the risk of leakage.
• Salt caverns can withstand high pressure and temperature variations, allowing for
faster filling and emptying of oil. Hence statement 2 is correct.
• Salt caverns are much faster and cheaper than excavating rock caverns and can be
done in flat or low-lying areas where salt deposits are found.
• Rock caverns are created by excavating hard rock formations, such as granite or basalt,
using explosives or mechanical methods.
• The process of solution mining is faster and simpler than excavating rock caverns,
which requires more time, labour, and equipment.
• They are suitable for storing natural gas, compressed air, and hydrogen. Hence
statement 3 is correct.
Source: https://indianexpress.com/article/explained/explained-economics/explained-
economics-oil-reserves-in-salt-caverns-8643242/

Q.24) Solution (c)


Statement Analysis:

• New Development Bank originated during the Sixth BRICS Summit in Fortaleza, Brazil.
• It is headquartered in Shanghai.
• Its key areas of operation are clean energy, transport infrastructure, irrigation,
sustainable urban development, and economic cooperation among the member
countries. Hence statement 1 is correct.
• BRICS is an acronym for the grouping of the world’s leading emerging economies,
namely Brazil, Russia, India, China, and South Africa.
• India is a founding member of BRICS. Hence statement 2 is correct.
• It conducted the 13th BRICS Summit in 2021 in New Delhi.
• The first BRICS Summit took place in 2009 in the Russian Federation. Hence statement
3 is correct.
• Recently, the 14th BRICS Summit was convened in 2022 in a virtual format.
Source: https://newsonair.gov.in/News?title=Prime-Minister-Modi-discusses-cooperation-
in-BRICS-with-South-African-President-Cyril-Ramaphosa-during-telephone-
conversation&id=462345

Q.25) Solution (c)


Statement Analysis:

• Green Hydrogen is produced by electrolysis using renewable energy.


• The electricity splits water into hydrogen and oxygen with by-products of water and
water vapor.
• It is called ‘green hydrogen’ because has no carbon footprint.
• Green Hydrogen as an energy source is seen as the next big thing as its usage would
lead to zero emissions.
• It is a clean-burning molecule, which can decarbonise a range of sectors including iron
and steel, chemicals, and transportation. Hence statement 1 is correct.
.
IASBABA’S AIPTS 2024 – TEST 4 (CURRENT AFFAIRS – JUNE 2023) SOLUTIONS

• It can be used in fuel cells to generate electricity, or power and heat. Hence statement
2 is correct.

Note:
• Brown hydrogen is produced using coal where the emissions are released into the air
• Blue hydrogen is produced from natural gas, where the emissions are captured using
carbon capture and storage.
Source: https://www.thehindu.com/news/national/kerala/anert-nominated-nodal-agency-
for-green-hydrogen-initiatives-in-state/article66957964.ece

Q.26) Solution (b)


Statement Analysis:

• The French Open is a major tennis tournament. Hence statement 1 is correct.


• It is one of the four Grand Slams in Tennis. Hence statement 2 is correct.
• It is also known as Roland-Garros.
• The tournament and venue are named after the French aviator Roland Garros.
• Roland Garros was a French aviation pioneer and fighter pilot. Hence statement 3 is
correct.
• It is scheduled for two weeks, beginning in late May each year at the Stade Roland
Garros in Paris, France
• The French Open is the premier clay court championship in the world.

Note:
• The Grand Slam tournaments consist of the following:
• Australian Open: in mid-January,
• French Open or Roland Garros: around late May,
• Wimbledon: in June end, and
• US Open tennis: at the end of August.
• The four annual tennis tournaments are collectively called the Grand Slam, also known
as majors.
• It is operated by the International Tennis Federation (ITF).
Source: https://newsonair.gov.in/News?title=French-Open-2023%3a-Novak-Djokovic-
defeats-Casper-Ruud%2c-creates-history-by-becoming-1st-man-to-reach-coveted-23-grand-
slam-title-mark-in-Paris&id=462359

Q.27) Solution (a)


Statement Analysis:

• The First Loss Default Guarantee (FLDG) is also known as the default loss guarantee
(DLG).
• FLDG is a lending model between a fintech and a regulated entity.
• Financial technology (fintech) is a new technology that seeks to improve and
automate the delivery and use of financial services. Hence statement 1 is correct.
• The credit risk is borne by the Lending service providers (LSPs) without having to
maintain any regulatory capital.
.
IASBABA’S AIPTS 2024 – TEST 4 (CURRENT AFFAIRS – JUNE 2023) SOLUTIONS

• The loan portfolio backed by FLDG is akin to the off-balance sheet portfolio of the
LSP wherein the nominal loans sit in the books of the lender without having to partake
in any lending process.
• FLDG helps expand the customer base of traditional lenders but relies on the fintech’s
underwriting capabilities. Hence statement 2 is correct.
• It is a safety-net arrangement among banks, non-banking finance companies (NBFC),
and lending service providers (LSPs), popularly known as fintech players) in the digital
lending space. Hence statement 3 is incorrect.
• Under this, a third party guarantees to compensate up to a certain percentage of
default.
• The fintech originates a loan and promises to compensate the partners up to a pre-
decided percentage in case customers fail to repay.

Note: Lending service providers (LSPs) are new-age players who use technology platforms in
the lending space. They are agents of a bank or NBFC. They carry out one or more of a lender’s
functions (in part or full) in customer acquisition, underwriting support, pricing support,
disbursement, servicing, monitoring, and recovery of specific loans or loan portfolios on
behalf of REs as per the outsourcing guidelines of the RBI.
Source: https://indianexpress.com/article/explained/explained-economics/rbi-permits-loan-
default-guarantee-in-digital-lending-to-boost-fintech-activity-8654538/

Q.28) Solution (c)


Statement Analysis:

• The World Food Programme (WFP) is the leading humanitarian organization saving
lives and changing lives, delivering food assistance in emergencies, and working with
communities to improve nutrition and build resilience. It was founded in 1961.
• It was founded by the Food and Agriculture Organization (FAO) and United Nations
General Assembly (UNGA). Hence statement 1 is correct.
• It is headquartered in Rome, Italy. Hence statement 2 is correct.
• The WFP has no independent source of funds; it is funded entirely by voluntary
donations. Hence statement 3 is correct.

Q.29) Solution (a)


Statement Analysis:

• Eon is a term in Earth science for the longest periods of time.


• Proterozoic Eon was the last eon of the Precambrian supereon.
• The Precambrian era spans Era (4500–544 Ma) and includes almost 90% of Earth’s
history.

Major Events in the Proterozoic Eon:


• Bacteria began producing oxygen, leading to the sudden rise of life forms.
• Eukaryotes (have a nucleus), emerge, including some forms of soft-bodied
multicellular.
• Earlier forms of fungi formed around this time. Hence statement 1 is correct.
.
IASBABA’S AIPTS 2024 – TEST 4 (CURRENT AFFAIRS – JUNE 2023) SOLUTIONS

• The early and late phases of this eon may have undergone Snowball Earth periods.
• Snowball Earth periods are when Earth suffered below-zero temperatures, extensive
glaciation, and as a result drop in sea levels. Hence statement 2 is correct.
• It was a very tectonically active period in the Earth’s history. Hence statement 3 is
incorrect.
• It featured the first definitive supercontinent cycles and modern mountain buildings.
• It is believed that 43% of modern continental crust was formed in the Proterozoic,
39% formed in the Archean, and only 18% in the Phanerozoic.
• In the late Proterozoic (most recent), the dominant supercontinent was Rodinia.
Source: https://indianexpress.com/article/technology/science/molecular-fossils-primordial-
life-8652075/

Q.30) Solution (c)


Statement Analysis:

• The Mission of Advanced and High-Impact Research (MAHIR) aims to facilitate


indigenous research, development, and demonstration of the latest and emerging
technologies in the power sector.
• It is planned for an initial period of five years from 2023-24 to 2027-28.
• It is launched by the Ministry of Power and the Ministry of New and Renewable
Energy. Hence statement 1 is correct.
• MAHIR operates through a two-tier structure comprising a Technical Scoping
Committee and an Apex Committee. Hence statement 2 is correct.
• It is led by the Chairperson of the Central Electricity Authority.

Objectives of MAHIR: -
• To identify emerging technologies in the power sector that have the potential for
future relevance globally.
• To serve as a platform for collective brainstorming.
• To Support pilot projects for indigenous technologies.
• Leverage foreign alliances to accelerate research and development efforts.
• To create a vibrant and innovative ecosystem in the power sector.
Source: https://pib.gov.in/PressReleaseIframePage.aspx?PRID=1930368

Q.31) Solution (c)


Statement Analysis:

• The Shangri-La Dialogue (SLD) is an inter-governmental security forum.


• Originally known as the Asia Security Summit was initiated in 2002 in response to the
evident need for a forum where Asia-Pacific defence ministers could engage in
dialogue aimed at building confidence and fostering practical cooperation.
• It is a Track 1 inter-governmental security forum. Hence statement 1 is correct.
• It is Asia’s largest annual security forum.
• It is held every year in Singapore since 2002. Hence statement 2 is incorrect.
• It is organised by the International Institute for Strategic Studies (IISS).
• IISS is a London-based independent think tank. Hence statement 3 is correct.
.
IASBABA’S AIPTS 2024 – TEST 4 (CURRENT AFFAIRS – JUNE 2023) SOLUTIONS

Note:
Different levels of Diplomacy:
• Track 1 Diplomacy: Official discussions typically involve high-level political and military
leaders.
• Track 2 Diplomacy: Unofficial dialogue and problem-solving activities involving
influential academic, religious, and NGO leaders and other civil society actors who can
interact more freely than high-ranking officials can.
• Track 3 Diplomacy: People-to-people diplomacy undertaken by individuals and private
groups.
Source: https://indianexpress.com/article/opinion/columns/india-us-defence-deal-pm-
modi-joe-biden-china-8649345/lite/

Q.32) Solution (b)


Statement Analysis:

• The National Food Safety Index is developed by FSSAI (Food Safety and Standards
Authority of India).
• The Food Safety and Standards Authority of India (FSSAI) is an autonomous statutory
body established under the Food Safety and Standards Act, 2006 (FSS Act).
• It is under the Ministry of Health & Family Welfare. Hence statement 1 is incorrect.
• The first National Food Safety Index was published in 2018-19.
• It was announced on the first-ever World Food Safety Day on 7th June 2019. Hence
statement 2 is correct.
• The World Health Organization (WHO) and the Food and Agriculture Organization of
the United Nations (FAO) jointly facilitate the observance of World Food Safety Day.
• The objective of the food safety index is to create a competitive and positive change
in the food safety ecosystem in the country.
• The Parameters include Human Resources and Institutional Data, Compliance, Food
Testing – Infrastructure and Surveillance, Training & Capacity Building, and Consumer
Empowerment.
Source: https://www.thehindu.com/news/national/kerala/state-comes-first-in-national-
food-safety-index/article66943095.ece

Q.33) Solution (b)


Statement Analysis:

• Financial Services Institution Bureau (FSIB) was established by passing a resolution by


the Cabinet Appointments Committee (ACC) to establish it in place of the Banks Board
Bureau (BBB). Hence statement 1 is incorrect.
• FSIB would be headed by a chairman, a central government nominee.
• The board would comprise the Secretaries of the DFS, the chairman of IRDAI, and a
deputy governor of the RBI.
• Additionally, it will have three part-time members who are experts in banking and
three more from the insurance sector. Hence statement 2 is correct.
• The FSIB selects the chiefs of public sector banks and insurance companies
.
IASBABA’S AIPTS 2024 – TEST 4 (CURRENT AFFAIRS – JUNE 2023) SOLUTIONS

• It has the mandate to issue guidelines and select general managers and
directors of state-run non-life insurers, general insurers, and Financial Institutions.
• It works under the Department of Financial Service, Ministry of Finance.
• It also monitors and assesses the performance of public sector banks, government-
owned financial institutions, and insurance companies. Hence statement 3 is correct.
Source: https://indianexpress.com/article/business/fsib-selects-new-chiefs-for-gic-re-nic-
8651319/

Q.34) Solution (b)


Statement Analysis:

• Kilauea volcano is located in Hawaii Volcanoes National Park on the southeastern part
of the island of Hawaii, U.S.A.
• It is one of the world’s most active volcanoes.
• It is an active shield volcano. Hence statement 1 is correct.
• Shield volcanoes: these are formed where a volcano produces low viscosity, runny
lava, spreading far from the source and forming a volcano with gentle slopes.
• It has erupted 34 times since 1952. From 1983 to 2018, it erupted almost
continuously.
• Statement 2 is also correct but not the correct explanation for statement 1.
Source: https://www.thehindu.com/news/international/kilauea-one-of-the-worlds-most-
active-volcanoes-begins-erupting-after-3-month-pause/article66944852.ece

Q.35) Solution (b)


Statement Analysis:

• World Meteorological Organisation (WMO) is a specialized agency of the United


Nations. Hence statement 1 is incorrect.
• International Meteorological Organization (IMO), founded in 1873 was its predecessor
organization.
• It is dedicated to meteorology (weather), climatology (climate), operational hydrology
(water), and other related geophysical sciences such as oceanography and
atmospheric chemistry.
• It has 192 Member States and Territories. India is a member of WMO.
• It is headquartered in Geneva, Switzerland. Hence statement 2 is correct.
• It releases reports such as the Greenhouse Gas Bulletin and Status of the World
Climate. Hence statement 3 is correct.

Note: Biparjoy was a deep depression formed over the southeast Arabian Sea,
which intensified into a cyclonic storm.
Source: https://www.downtoearth.org.in/news/natural-disasters/biporjoy-very-severe-
cyclone-over-central-arabian-sea-advancing-northwards-as-monsoon-arrives-in-kerala-
89902

Q.36) Solution (c)


.
IASBABA’S AIPTS 2024 – TEST 4 (CURRENT AFFAIRS – JUNE 2023) SOLUTIONS

Statement Analysis:

• The Gilgit manuscripts were discovered in the Naupur village (Gilgit region), Pakistan
Occupied Kashmir by an archaeologist Sir Aurel Stein.
• It was written between the 5th -6th centuries CE. Hence statement 1 is correct.
• It is the oldest surviving manuscript collection in Indian Subcontinent. Hence
statement 2 is correct.
• It was written on the birch bark folios documents written on pieces of an inner layer
of the bark of birch trees found in the Kashmir region.
• It contains both canonical and non-canonical Jain and Buddhist works that throw light
on the evolution of much religious-philosophical literature. Hence statement 3 is
correct.
Source: https://pib.gov.in/PressReleasePage.aspx?PRID=1931043

Q.37) Solution (d)


Statement Analysis:
Anak Krakatau Volcano is an island in a caldera situated in the Sunda Strait between the
islands of Java and Sumatra in Indonesia. It is part of the Ujung Kulon National Park, listed
in UNESCO’s World Heritage Site.
Hence option d is correct.

Source: https://indianexpress.com/article/world/indonesia-anak-krakatau-volcano-
eruption-8655907/

Q.38) Solution (b)


Statement Analysis:

• Kosovo is a small, landlocked country in the Balkans. Hence statement 1 is incorrect.


• It is bordered by Albania, North Macedonia, Montenegro, and Serbia. Hence
statement 2 is correct.
.
IASBABA’S AIPTS 2024 – TEST 4 (CURRENT AFFAIRS – JUNE 2023) SOLUTIONS

Source: https://www.thehindu.com/news/international/explained-why-is-there-trouble-in-
kosovo-again/article66939379.ece

Q.39) Solution (b)


Statement Analysis:

• NATO is a Western defensive military alliance led by the United States. Hence
statement 1 is correct.
• It is also called the Washington Treaty.
• It came into being after World War II as a counter to the Soviet Union’s possible
expansion attempts in Europe.
• It is headquartered in Brussels, Belgium. Hence statement 2 is correct.
• The Headquarters of Allied Command Operations is in Mons, Belgium.
• The U.S. contributes roughly three-fourths of NATO’s budget.
• Its members include Belgium, Canada, Denmark, France, Iceland, Italy, Luxembourg,
the Netherlands, Norway, Portugal, the United Kingdom, the United States, Greece,
Turkey, Germany, Spain, Czech, Hungary, Poland, Bulgaria, Estonia, Latvia, Lithuania,
Romania, Slovakia, Slovenia, Albania, Croatia, Montenegro, and North Macedonia.
• India is not a member of NATO. Hence statement 3 is incorrect.
.
IASBABA’S AIPTS 2024 – TEST 4 (CURRENT AFFAIRS – JUNE 2023) SOLUTIONS

Source: https://www.thehindu.com/news/international/explained-why-is-there-trouble-in-
kosovo-again/article66939379.ece

Q.40) Solution (c)


Statement Analysis:

• Ram Prasad Bismil was a revolutionary freedom fighter with a poet’s heart.
• Bismil was born on 11 June 1897, in a village in Uttar Pradesh’s Shahjahanpur district
to Murlidhar and Moolmati.
• British authorities hanged him for his involvement in the Kakori Train Action. He was
hanged in the Gorakhpur jail in 1927 and cremated on the banks of the Rapti River.
• Bismil joined Arya Samaj in 1875.
• He formed the organization Matrivedi with Genda Lal Dixit, a schoolteacher.
• He formed the Hindustan Republic Association in 1924.
• Hindustan Republic Association was a revolutionary party to fight against British
colonial rule in India.
• It was evolved into Hindustan Socialist Republican Association (HSRA) in 1928
by Chandrashekhar Azad, Bhagat Singh, and others. Hence statement 1 is correct.
• He published a pamphlet titled ‘Deshwasiyon ke Naam’ and distributed it along with
his poem ‘Mainpuri ki Pratigya’ in 1918. Hence statement 2 is correct.
• He also wrote the cult song “Mera rang de Basanti Chola”.
• He is a revered symbol of patriotism and Hindu-Muslim unity. Hence statement 3 is
correct.
• He attended the 1921 session of the Indian National Congress at Ahmedabad.
Source: https://indianexpress.com/article/explained/explained-history/a-ram-prasad-bismil-
revolutionary-poet-8656213/

Q.41) Solution (a)


.
IASBABA’S AIPTS 2024 – TEST 4 (CURRENT AFFAIRS – JUNE 2023) SOLUTIONS

Statement Analysis:

• Nutri garden is a method of planting and harvesting nutrient-rich crops in residential


houses or in their vicinity to meet the requirements of the family all year round. Hence
statement 1 is correct.
• It is a cost-effective model to grow nutrient-rich crops for personal or community
consumption to promote good health and well-being.
• It increases the availability of food and nutrient sources. Hence statement 2 is
incorrect.
• It can act as a source of supplementary income.
• The crops harvested are Fresh and Safe (chemical-free).
• It helps tackle both under-nutrition and over-nutrition by adopting a sustainable life
cycle approach.
Source: https://timesofindia.indiatimes.com/city/kochi/nutri-garden-project-a-
success/articleshow/100926238.cms?from=mdr

Q.42) Solution (a)


Statement Analysis:

• INS Trishul is the second frigate of the Talwar class of the Indian Navy. Hence
statement 1 is correct.
• The guided-missile frigate joined the arsenal of the Indian Navy in 2003.
• It is built in Russia under an Indo-Russian joint production. Hence statement 2 is
incorrect.
• These missile frigates are modified Krivak III class frigates from Russia.
Source: https://www.hindustantimes.com/india-news/ins-trishul-in-durban-to-mark-130th-
anniversary-of-mahatma-gandhi-s-satyagraha-101686130494752.html

Q.43) Solution (c)


Statement Analysis:

• Homo Naledi is an extinct human species that lived hundreds of thousands of years
ago. Hence statement 1 is incorrect.
• Homo Naledi is a species of human discovered in the Rising Star cave system, a
UNESCO World Heritage Site in South Africa in 2013. The Cradle of Humankind World
Heritage Site, located in South Africa, has been vital to the study of human origins. In
recent decades, Berger and his team have located and documented more hominin
fossils than any other exploration program in the region.
• Homo naledi exhibits a combination of primitive and modern features and is not a
direct ancestor of modern humans.
• They are believed to have existed between 300,000 to 200,000 years in Southern
Africa.
• They walked upright.
• Their shoulders were built for climbing.
• Their teeth were shaped like that of older primates.
.
IASBABA’S AIPTS 2024 – TEST 4 (CURRENT AFFAIRS – JUNE 2023) SOLUTIONS

Note- The research team has discovered the remains of Homo naledi adults and children that
were laid to rest in the fetal position within cave depressions and covered with soil. The
burials are older than any known Homo sapiens burials by at least 100,000 years.

Source: https://indianexpress.com/article/technology/science/homo-naledi-long-lost-
human-species-buried-their-dead-and-carved-cave-symbols-say-scientists-8648488/

Q.44) Solution (b)


Statement Analysis:

• The Securities Appellate Tribunal (SAT) is a statutory body created under the
provisions of the SEBI Act, 1992. Hence statement 1 is incorrect.
• It has jurisdiction throughout India.
• It consists of a Presiding Officer and two other members.
• The Presiding Officer is appointed by the Central Government in consultation with
the Chief Justice of India or his nominee. Hence statement 2 is correct.
• It has powers similar to a civil court. Hence statement 3 is correct.
• The appeals from its orders can be challenged in the Supreme Court.
Source: https://www.business-standard.com/markets/news/relief-for-brickwork-ratings-sat-
sets-aside-sebi-order-cancelling-licence-123060600575_1.html

Q.45) Solution (c)


Statement Analysis:

• National Highways Authority of India (NHAI) was constituted by an Act of Parliament


in 1988 under the administrative control of the Ministry of Road Transport and
Highways NHAI has been set up as a Central Authority to develop, maintain and
manage the National Highways entrusted to it by the Government of India. It is
an autonomous organization that looks after the management of the complete
network of National Highways in the country.
• It works under the Ministry of Road Transport and Highways. Hence statement 1 is
correct.
• It is headquartered in New Delhi.
• It has Chairman, and not more than five full-time Members and four part-time
Members.
• They are appointed by the Central Government. Hence statement 2 is correct.
Source: https://www.hindustantimes.com/cities/chandigarh-news/nhai-shifts-toll-plaza-
from-jagjitpura-to-new-location-on-jalandhar-barnala-stretch-amidst-farmer-protests-
101685895270477.html

Q.46) Solution (a)


Statement Analysis:

• Prepaid Payment Instruments (PPIs) are instruments that facilitate the purchase of
goods and services.
.
IASBABA’S AIPTS 2024 – TEST 4 (CURRENT AFFAIRS – JUNE 2023) SOLUTIONS

• These include financial services, remittance facilities, etc. Hence statement 1 is


correct.
• Prepaid instruments can be issued as smart cards, magnetic stripe cards, internet
accounts, internet wallets, mobile accounts, mobile wallets, paper vouchers, and any
of the instruments used to access the prepaid amount. Hence statement 2 is
incorrect.
Source: https://indianexpress.com/article/explained/explained-economics/deposit-
insurance-cover-for-ppis-how-will-customers-benefit-8648010/

Q.47) Solution (c)


Statement Analysis:

• Van Dhan Vikas Kendras are set up under the Pradhan Mantri VanDhan Yojana
(PMVDY). Hence statement 1 is correct.
• They aim to promote Minor Forest Products (MFPs) -centric livelihood development
of tribal gatherers and artisans.
• Minor forest produce are economic commodities growing naturally in a forest and
sold for purposes other than timber and fuel. Examples include bamboo, wild honey,
gum, lac, waxes, resins, etc. Hence statement 2 is correct.
• These Kendras would act as common facility centres for procurement cum value
addition to locally available MFPs.

Note:
Pradhan Mantri VanDhan Yojana (PMVDY) was launched in 2018. It is an initiative of
the Ministry of Tribal Affairs and TRIFED. Its objective is to improve tribal incomes through
the value addition of tribal products. It is a Market Linked Tribal Entrepreneurship
Development Program for forming clusters of tribal Self-Help-Groups (SHGs) and
strengthening them into Tribal Producer Companies.
Source: https://www.downtoearth.org.in/news/forests/these-odisha-tribal-women-earn-
big-with-mahua-ladoos-89825

Q.48) Solution (d)


Statement Analysis:

• District Legal Services Authority is a statutory body that is formed in the various states
of India under the Legal Services Authorities Act, of 1987. Hence statement 1 is
correct.
• Its objective is to provide free legal aid and services to the weaker sections of this
society to make sure that opportunities for securing justice are not denied to any
citizen because of economic or other disabilities.
• It is organized at the district level to provide effective monitoring of legal aid
programmes and their composition.
• The State Government in consultation with the Chief Justice of the High Court
constitutes it. Hence statement 2 is correct.
• District Judge is appointed as its chairman. Hence statement 3 is correct.
.
IASBABA’S AIPTS 2024 – TEST 4 (CURRENT AFFAIRS – JUNE 2023) SOLUTIONS

• Other members must have the experience and qualifications as prescribed by the
State Government.
• The Government in consultation with the Chief Justice of the High Court may nominate
these members.
• The person to be appointed shall belong to the State Judicial Service and not lower
than rank than that of a Subordinate Judge or Civil Judge posted at the seat of the
District Judiciary as Secretary of the District Authority.
Source: https://pib.gov.in/PressReleseDetailm.aspx?PRID=1930032

Q.49) Solution (a)


Statement Analysis:

• The National Institutional Ranking Framework (NIRF) is a methodology to rank


institutions of higher education in India. Hence statement 1 is correct.
• Its objective is to help universities to improve their performance on various ranking
parameters and identify gaps in research and areas of improvement.
• It is released by the Ministry of Human Resource Development (MHRD). Hence
statement 2 is incorrect.
• Five broad categories of parameters identified in the NIRF are Teaching, learning, and
resources (TLR); research and professional practice; graduation outcome; outreach;
and inclusivity and perception.
Source: https://indianexpress.com/article/education/nirf-ranking-2023-live-updates-top-10-
educational-institutes-released-iit-madras-iisc-bangalore-aiims-delhi-nirfindia-org-8644726/

Q.50) Solution (c)


Statement Analysis:

• The Agra Fort has been occupied by several dynasties, including the Rajputs, Mughals,
Jats, and Marathas. Hence statement 1 is correct.
• The fort underwent construction for eight years under the reign of Akbar in 1565.
• However, the Archaeological Survey of India (ASI) lacks knowledge about who built
the Agra Fort first and what changes it underwent during Akbar’s reign.
• It is made of red sandstone.
• It became a UNESCO World Heritage site in 1983. Hence statement 2 is correct.
• It comprises the Jahangir Palace and the Khas Mahal (built by Shah Jahan), Diwan-i-
Khas, and two very beautiful mosques. Hence statement 3 is correct.
• It was the main residence of the emperors of the Mughal Dynasty until 1638 when the
capital was shifted from Agra to Delhi.
Source: https://www.indiatoday.in/india/story/asi-does-not-know-built-agra-fort-2388907-
2023-06-05

Q.51) Solution (a)

Statement Analysis:
.
IASBABA’S AIPTS 2024 – TEST 4 (CURRENT AFFAIRS – JUNE 2023) SOLUTIONS

• Ethanol is an agricultural by-product that is mainly obtained from the processing of


sugar from sugarcane, but also from other sources such as rice husk or maize.
• Blending ethanol with petrol to burn less fossil fuel while running vehicles is
called Ethanol Blending.
• E20 fuel is a blend of 20% ethanol and 80% petrol. Hence statement 1 is incorrect.
• E20 fuel reduces vehicular tailpipe emissions by lowering carbon monoxide,
hydrocarbons, and nitrogen oxide levels.
• It improves engine performance and reduces maintenance costs by preventing
corrosion and deposits. Hence statement 2 is correct.
• It reduces the import bill for crude oil by substituting domestic ethanol production.
• It is estimated that a 5% blending (105 crore litres) can result in the replacement
of around 1.8 million barrels of crude oil. Hence statement 3 is incorrect.
Source: https://www.downtoearth.org.in/news/energy/private-sector-aggressively-
pursuing-production-of-green-hydrogen-hardeep-singh-puri-at-g20-event-90039

Q.52) Solution (c)

Statement Analysis:

• The new variety's Cry2Ai gene confers pest resistance on the cotton and is also derived
from the bacterium Bacillus thuringiensis.
• While Cry2Ai targets a wider variety of insects, including some coleopteran (beetle)
and dipteran (fly) species, Cry1Ac primarily targets lepidopteran insects (moths and
butterflies). Hence option c is correct.
Source: https://www.thehindu.com/sci-tech/agriculture/explained-what-is-the-status-of-
transgenic-crops-in-india/article66968448.ece/amp/

Q.53) Solution (b)

Statement Analysis:

• SGBs are issued by the Reserve Bank of India on behalf of the Government of India.
Hence statement 1 is correct.
• The main objective of the SGB Scheme is to mobilize the gold held by households and
institutions in the country, reduce reliance on the import of gold over time to meet
the domestic demand, and promote FDI in the gold and jewellery sector. Hence
statement 2 is correct.
• The tenure of the SGB will be for a period of eight years with an option of premature
redemption after 5th year. Hence statement 3 is incorrect.
• The minimum permissible investment will be one gram of gold and the maximum limit
of subscription shall be 4 Kg for individuals, 4 Kg for HUF, and 20 Kg for trusts and
similar entities per fiscal year (April-March) notified by the Government from time to
time.
.
IASBABA’S AIPTS 2024 – TEST 4 (CURRENT AFFAIRS – JUNE 2023) SOLUTIONS

• SGBs will be restricted for sale to resident individuals, HUFs (Hindu Undivided Family),
trusts, universities, and charitable institutions.
Source: https://pib.gov.in/PressReleaseIframePage.aspx?PRID=1932437

Q.54) Solution (b)

Statement Analysis:

• Global Wind Day is an annual event since 2007 to promote wind energy as a clean and
renewable source of power. Hence statement 1 is incorrect.
• It was started by the European Wind Energy Association (EWEA) and later joined by
the Global Wind Energy Council (GWEC).
• GWEC is a member-based organisation that represents the entire wind energy sector.
• Global Wind Day 2023 was celebrated on 15th June 2023 by the Ministry of New and
Renewable Energy (MNRE) with the theme of “Pawan - Urja: Powering the Future of
India”. Hence statement 2 is correct.
• MNRE has set the target of 500 GW renewable energy capacity by 2030 and Wind
Atlas at 150 meters above ground level was also launched by the National Institute of
Wind Energy (NIWE), estimating the onshore wind potential at 1,164 GW.
Source: https://pib.gov.in/PressReleaseIframePage.aspx?PRID=1932715

Q.55) Solution (d)

Statement Analysis:

• The United States has entered into a landmark security pact with Papua New Guinea.
• The agreement, which allows the U.S. military to develop and operate bases in Papua
New Guinea, grants access to strategic ports and airports, including the Lombrum
Naval Base on Manus Island and facilities in Port Moresby. Hence option d is correct.

Source: https://www.thehindu.com/news/international/us-signs-new-security-pact-with-
papua-new-guinea-amid-competition-with-china/article66881298.ece
.
IASBABA’S AIPTS 2024 – TEST 4 (CURRENT AFFAIRS – JUNE 2023) SOLUTIONS

Q.56) Solution (a)

Statement Analysis:

• The Central Bureau of Investigation (CBI) was established by a resolution of the


Ministry of Home Affairs and later transferred to the Ministry of Personnel, Public
Grievances, and Pensions, currently functioning as an attached office. Its
establishment was recommended by the Santhanam Committee on the Prevention of
Corruption.
• The CBI operates under the Delhi Special Police Establishment (DSPE) Act of 1946. It
is neither a constitutional nor a statutory body. It investigates cases related to bribery,
governmental corruption, breaches of central laws, multi-state organized crime, and
multi-agency or international cases.
• The consent of the state government to the CBI can be either case-specific or general.
Hence statement 1 is correct.
• General consent is normally given by states to help the CBI in the seamless
investigation of cases of corruption against central government employees in their
states. This is essentially consent by default, which means CBI may begin
investigations taking consent as having been already given.
• In the absence of general consent, CBI would have to apply to the state
government for its consent in every individual case, and before taking even small
actions.
• Mizoram, West Bengal, Chhattisgarh, Rajasthan, Maharashtra, Kerala, Jharkhand,
Punjab, Tamil Nadu, and Meghalaya have withdrawn their general consent given to
the CBI.
• The states can withdraw the general consent given to the Central Bureau of
Investigation (CBI), under Section 6 of the Delhi Special Police Establishment (DSPE)
Act, 1946. Hence statement 2 is incorrect.
Source: https://www.thehindu.com/news/national/tamil-nadu/tamil-nadu-withdraws-
general-consent-for-cbi-within-the-
state/article66969381.ece#:~:text=The%20DMK%2Dled%20government%20in,its%20permis
sion%20in%20the%20State

Q.57) Solution (b)

Statement Analysis:

• Cargo release time is defined as the time taken from the arrival of the cargo at
the customs station to its out-of-charge. Hence statement 1 is correct.
• For domestic clearance in case of imports and arrival of the cargo at the customs
station to the eventual departure of the carrier in case of exports.
.
IASBABA’S AIPTS 2024 – TEST 4 (CURRENT AFFAIRS – JUNE 2023) SOLUTIONS

• Cargo release time is a key indicator of trade efficiency and ease of doing business, as
it reflects the effectiveness of customs procedures and other regulatory processes
involved in cross-border trade. Hence statement 2 is correct.
• Cargo release time is measured using Time Release Study (TRS), a performance
measurement tool recommended by the World Customs Organization (WCO). Hence
statement 3 is incorrect.

Note:
• The World Customs Organization (WCO) is an independent intergovernmental
body whose mission is to enhance the effectiveness and efficiency of customs
administrations.
• It is the only international organization with competence in customs matters which
can rightly call itself the voice of the international customs community.
• It has its headquarters in Brussels, Belgium.
Source: https://pib.gov.in/PressReleseDetail.aspx?PRID=1932587

Q.58) Solution (d)

Statement Analysis:

• The Central Board of Indirect Taxes and Customs (CBIC) has released the National
Time Release Study (NTRS) Report 2023.
• It measures the cargo release time at various ports in India. Hence option d is correct.
• The report aims to assess the progress made towards the National Trade Facilitation
Action Plan (NTFAP) targets, identify the impact of various trade facilitation initiatives,
and identify the challenges to a more expeditious reduction in release time.
Source: https://pib.gov.in/PressReleseDetail.aspx?PRID=1932587

Q.59) Solution (c)

Statement Analysis:

• The Oslo Forum is a series of retreats for international conflict mediators, high-level
decision-makers, and other peace process actors.
• It provides a discreet and informal space to reflect on current mediation practices,
collaborate across institutional and conceptual divides, and advance negotiations.
Hence statement 1 is correct.
• The Forum is co-hosted by the Royal Norwegian Ministry of Foreign Affairs and the
Centre for Humanitarian Dialogue (HD).
• The Centre for Humanitarian Dialogue (HD) is a private diplomacy organization that
specializes in mediation and dialogue processes to prevent and resolve armed
conflicts.
.
IASBABA’S AIPTS 2024 – TEST 4 (CURRENT AFFAIRS – JUNE 2023) SOLUTIONS

• HD was established in 1999, based in Geneva, Switzerland, and operates globally.


Hence statement 2 is incorrect.
• All discussions are confidential and take place under the Chatham House Rule.
• Chatham House (UK) is an independent policy institute and a forum for debate and
dialogue.
• When a meeting is held under the Chatham House Rule, participants are free to use
the information received, but neither the identity nor the affiliation of the speaker(s),
nor that of any other participant, may be revealed. Hence statement 3 is correct.
• Participation in this forum is by invitation only. This is the first time India and Pakistan
were invited to the talks.

Source: https://www.thehindu.com/news/national/in-a-first-india-and-pakistan-special-
envoys-participate-in-western-talks-with-taliban-in-
norway/article66972071.ece#:~:text=The%20talks%2C%20which%20came%20in,similar%20
efforts%20at%20the%20Moscow

Q.60) Solution (a)

Statement Analysis:

• Saksham Anganwadi and Poshan 2.0 is a centrally sponsored scheme implemented by


states and union territories. Hence statement 1 is correct.
.
IASBABA’S AIPTS 2024 – TEST 4 (CURRENT AFFAIRS – JUNE 2023) SOLUTIONS

• It seeks to address the challenges of malnutrition in children, adolescent girls,


pregnant women, and lactating mothers through a strategic shift in nutrition content
and delivery and by the creation of a convergent ecosystem to develop and promote
practices that nurture health, wellness, and immunity. Hence statement 2 is correct.
• It provides a package of six services, namely, supplementary nutrition, pre-school non-
formal education, nutrition and health education, immunization, health check-up, and
referral services.
• It provides services to all eligible beneficiaries, namely, children in the age group of 0-
6 years, pregnant women, and lactating mothers through the platform of Anganwadi
Centres across the country.
• It is being implemented by the Ministry of Women and Child Development. Hence
statement 3 is incorrect.
Source: https://www.thehindu.com/news/cities/Delhi/app-allows-migrant-workers-to-
access-nurseries-wherever-they-live/article66961644.ece

Q.61) Solution (b)

Statement Analysis:

• The National Cooperative Union of India (NCUI) is the apex body representing all
sectors of the Indian co-operative movement. Hence statement 1 is correct.
• Its objectives include the promotion and development of the co-operative movement
in India, to educate, guide and assisting the people in their efforts, and to build up and
expand the co-operative sector.
• Its membership is open to national and state-level co-operative organisations as well
as multi-state co-operative societies. Hence statement 2 is incorrect.
• As of 2016, NCUI had 260 members, including, 17 National, 163 State, and 80 Multi-
State Cooperatives.
• It is organising Indian Cooperative Congress (ICC) with the theme of Amrit Kaal:
Prosperity through Cooperation for a Vibrant India. Hence statement 3 is correct.
• It will be chaired by the Union Minister for Home and Cooperation.
• The objectives of the Indian Cooperative Congress are:
✓ To discuss various trends in the cooperative movement.
✓ To showcase the best practices being adopted by the successful cooperatives.
✓ To deliberate on challenges being faced by the cooperative world.
Source: https://newsonair.gov.in/News?title=PM-Narendra-Modi-to-inaugurate-17th-
Indian-Cooperative-Congress-in-New-Delhi&id=463350

Q.62) Solution (d)

Statement Analysis:
.
IASBABA’S AIPTS 2024 – TEST 4 (CURRENT AFFAIRS – JUNE 2023) SOLUTIONS

• The National Research Foundation (NRF) is an apex body to promote, fund, and
mentor scientific research in higher education institutions.
• The initial budget of Rs 50,000 crore for over a five-year period between 2023 and
2028.
• It will provide fellowships for post-doctoral research, the funding for which is not
available right now. Hence statement 1 is correct.
• It will be administratively housed in the Department of Science and Technology (DST).
Hence statement 2 is correct.
• It would have a 16-member governing board. Of these two members from DST, five
from industry, one from humanities, and six experts would be selected depending on
the nature of the project being evaluated.
• The Prime Minister will be the ex-officio president of the NRF. The Minister of Science
and Technology and the Minister of Education will be the ex-officio vice presidents.
Hence statement 3 is correct.
• The NRF’s functioning will be governed by an executive council chaired by the
Principal Scientific Advisor to the government of India.
• The NRF will forge collaborations among the industry, academia, government
departments, and research institutions. Hence statement 4 is correct.
• It will create an interface mechanism for the participation and contribution of
industries and state governments in addition to the scientific and line ministries.
Source: https://newsonair.gov.in/News?title=Centre-gives-nod-to-National-Research-
Foundation-Bill%2c-2023%3b-50-thousand-crore-rupees-to-be-allocated-to-strengthen-
country%26%2339%3bs-research-ecosystem&id=463341

Q.63) Solution (c)

Statement Analysis:

• Padma Awards was instituted in 1954 to recognize achievements in all fields of


activities where an element of public service is involved. Hence statement 1 is correct.
• The Awards are given in three categories:
✓ Padma Vibhushan: for exceptional and distinguished service.
✓ Padma Bhushan: for distinguished service of higher order.
✓ Padma Shri: for distinguished service.
• It is announced every year on the occasion of Republic Day and is presented by the
President of India. Hence statement 2 is incorrect.
• There was a brief interruption(s) during the years: 1978 and 1979 and 1993 to 1997.
• All persons without distinction of race, occupation, position, or sex are eligible for
these awards.
• Government servants including those working with PSUs, except doctors and
scientists, are not eligible for these Awards. Hence statement 3 is correct.
.
IASBABA’S AIPTS 2024 – TEST 4 (CURRENT AFFAIRS – JUNE 2023) SOLUTIONS

• The awardees are presented a Sanad (certificate) signed by the President and a
medallion.
• The recipients are also given a small replica of the medallion, which they can wear
during any ceremonial/State functions etc.
Source: https://newsonair.gov.in/News?title=MHA-announced-online-nominations-for-
Padma-Awards-2024-to-open-till-15th-September&id=463330

Q.64) Solution (a)

Statement Analysis:

• Champions Portal is a single-window grievance redressal portal for Micro, Small &
Medium Enterprises(MSME). Hence statement 1 is correct.
• The CHAMPIONS stands here for the Creation and Harmonious Application of Modern
Processes for Increasing Output and National Strength.
• It is created in a Hub & Spoke Model.
• The hub-and-spoke model is a transportation or distribution system that uses a central
hub or node as a focal point for the movement of goods, people, or information to
and from various spokes or peripheral locations.
• Hub is situated in New Delhi in the Secretary MSME’s office.
• Spokes will be in the States in various offices and institutions of the Ministry.
• It was launched on June 1, 2020, by the Ministry of Micro, Small & Medium
Enterprises. Hence statement 2 is incorrect.
• The Champions 2.0 Portal will now incorporate AI-driven chatbots.
• It will be available in 11 languages, including Hindi, Gujarati, Bengali, and Kannada.
• The portal will also have a real-time feedback mechanism for its analysis.
Source: https://pib.gov.in/PressReleasePage.aspx?PRID=1935748

Q.65) Solution (b)

Statement Analysis:

• The word ‘Kharchi’ is derived from two Tripuri words–‘khar’ or kurta meaning sin and
‘chi’ or si meaning cleaning.
• Kharchi Puja is one of the main festivals of Tripura. Hence statement 1 is incorrect.
• It is performed during the months of July-August on the eighth day of the new moon.
Hence statement 2 is correct.
• During the festival, the people of Tripura worship the earth along with their 14 deities.
• The fourteen Gods are worshipped by the Royal priest ‘Chantai.
• It lasts for seven days and it takes place at old Agartala in the Fourteen Gods
temple known as the ‘Chaturdasha Devata’ temple premises.
• The Kharchi Puja deities do not have a full body.
.
IASBABA’S AIPTS 2024 – TEST 4 (CURRENT AFFAIRS – JUNE 2023) SOLUTIONS

• They have only heads that are worshipped.


• On the day of the puja, the fourteen Gods are taken from the temple to the river
Saidra by the Chantai members and given to bathe with the holy river water.
• They are then carried back to the temple.
• The festival is celebrated by both tribal and non-tribal people of Tripura.
Source: https://indianexpress.com/article/lifestyle/art-and-culture/kharchi-puja-2023-date-
history-significance-rituals-8688105/lite/

Q.66) Solution (a)

Statement Analysis:

The Government of India, through the Ministry of Mines, is set to unveil for the first time “the
list of Critical Minerals for India”.
• Minerals such as antimony, cobalt, gallium, graphite, lithium, nickel, niobium, and
strontium are among the 22 assessed to be critical for India. Hence statement 1 is
correct.
• Critical minerals are elements that are crucial to modern-day technologies and are at
risk of supply chain disruptions. Hence statement 2 is correct.
• Many of these are required to meet the manufacturing needs of green technologies,
high-tech equipment, aviation, and national defence.
Source: https://pib.gov.in/PressReleseDetailm.aspx?PRID=1936083

Q.67) Solution (c)

Statement Analysis:

• National Statistics Day is celebrated on 29th June every year. Hence statement 1 is
correct.
• This year, the theme selected for Statistics Day, 2023 is ‘Alignment of State Indicator
Framework with National Indicator Framework for Monitoring Sustainable
Development Goals’.
• It is celebrated to commemorate the birth anniversary of Prof. Prasanta Chandra
Mahalanobis. Hence statement 2 is incorrect.
• Prasanta Chandra Mahalanobis is considered the father of modern statistics in India.
• He founded the Indian Statistical Institute (ISI) and shaped the Planning Commission.
Source: https://newsonair.gov.in/News?title=NSSO-in-association-with-S.-K.-Somaiya-
College-in-Mumbai%2c-celebrates-17th-Statistics-Day-today&id=463398

Q.68) Solution (d)

Statement Analysis:
.
IASBABA’S AIPTS 2024 – TEST 4 (CURRENT AFFAIRS – JUNE 2023) SOLUTIONS

• The Central Board for Film Certification (CBFC) is a statutory body under
the Cinematograph Act 1952. Hence statement 1 is correct.
• It is headed by a central government-appointed chairperson and 12-45 non-official
members.
• The members are eminent persons from social science, education, law, arts, or films
background appointed.
• It works under the Ministry of Information and Broadcasting. Hence statement 2 is
correct.
• It is headquartered in Mumbai, Maharashtra.
• Films can be publicly exhibited in India only after they are certified by the CBFC. Hence
statement 3 is correct.
• It examines films for content that may be harmful or unsuitable for some particular
audiences, particularly children and young people.
• It ensures that films adhere to ethical standards, respecting cultural values and
societal norms.
• After evaluating the content and classifying the film, the CBFC grants a certificate that
permits the film’s public exhibition.

Note: Films are certified under four categories:-


• “U”: unrestricted public exhibition.
• “A”: restricted to adult audiences.
• “U/A”: unrestricted public exhibition subject to parental guidance for children below
the age of twelve.
• “S”: restricted to specialized audiences such as doctors or scientists. The board may
also refuse to certify.

Source: https://newsonair.gov.in/News?title=72-Hoorain-was-granted-
%e2%80%98A%e2%80%99-certification-in-2019%2c-clarifies-CBFC&id=463392

Q.69) Solution (c)

Statement Analysis:

• The Indian Council of Forestry Research and Education (ICFRE) will be the
administrator of the Green Credit Programme (GCP). Hence statement 1 is correct.
• It will manage, monitor and operate the entire programme.
• ICFRE is an autonomous organisation or governmental agency under the MoEFCC. It
aims to generate, advance, and disseminate scientific knowledge and technologies for
ecological security, improved productivity, livelihood enhancement, and sustainable
use of forest resources through forestry research and education.
.
IASBABA’S AIPTS 2024 – TEST 4 (CURRENT AFFAIRS – JUNE 2023) SOLUTIONS

• Its objective is to create a market-based mechanism for undertaking environment-


friendly activities such as afforestation programmes, and water conservation by
providing additional incentives in the form of green credits. Hence statement 2 is
correct.
• It will be launched at the national level to incentivize voluntary environmental
actions of various stakeholders.
• It will encourage private sector industries by taking actions, which are able to
converge with activities relevant to generating or buying Green Credits.

Note: Green Credit is an incentive that individuals, farmer-producer organisations (FPO),


industries, and rural and urban local bodies, among other stakeholders, will be able to earn
for environment-positive actions.
Source: https://www.downtoearth.org.in/news/governance/centre-proposes-draft-rules-
for-green-credit-programme-90304

Q.70) Solution (a)

Statement Analysis:

Senegal has become the fourth country after South Africa, Indonesia, and Vietnam to sign the
JET-P deal.
• Just Energy Transition Partnership (JET-P) Deal is an initiative to accelerate the phasing
out of coal and reduce emissions. Hence statement 1 is correct.
• Its objective is to reduce emissions in the energy sector and accelerate the coal phase-
out process.
• It is modelled for South Africa, to support South Africa’s decarbonisation efforts.
• The JETP was launched at the 26th UN Climate Change Conference of the Parties
(COP26) in Glasgow with the support of the United Kingdom (UK), the United States
(US), France, Germany, and the European Union (EU). Hence statement 2 is incorrect.
Source: https://www.downtoearth.org.in/news/africa/senegal-signs-just-energy-transition-
partnership-deal-after-south-africa-indonesia-vietnam-90290

Q.71) Solution (b)

Statement Analysis:

• Provident Fund (PF) is a fixed amount of money that is contributed by employees from
their salary until retirement. Hence statement 1 is correct.
• It is used to provide a lump sum or monthly payments to salaried employees when
they retire.
• It provides good investment opportunities to individuals upon retirement. Hence
statement 2 is correct.
.
IASBABA’S AIPTS 2024 – TEST 4 (CURRENT AFFAIRS – JUNE 2023) SOLUTIONS

• The one-time inflow of money can aid them in changing homes, opening new
businesses, educating dependents, medical emergencies or just investing in mutual
funds.
• The government regulates provident funds in India through the Employees’ Provident
Fund Organization (EPFO). Hence statement 3 is incorrect.
• EPFO implements the Employees’ Provident Fund and Miscellaneous Provisions Act,
of 1952.
Source: https://www.thehindu.com/news/national/epfo-extends-deadline-to-apply-for-
higher-pension-till-july-11/article67012846.ece

Q.72) Solution (c)

Statement Analysis:

• The United Nations Office for Outer Space Affairs (UNOOSA) helps countries use space
data and technologies to prevent and manage disasters. Hence statement 1 is correct.
• Its objective is to help all countries, especially developing countries, access and
leverage the benefits of space to accelerate sustainable development.
• It helps countries understand the fundamentals of international space law and
increase their capacity to draft or revise national space law and policy.
• It supports transparency in space activities.
• It helps to promote sustainable development through space.
• It is headquartered in Vienna, Austria. Hence statement 2 is incorrect.
• It is headed by an Office of the Director and has five sections
• The Office of the Director (OD) oversees and coordinates the strategic direction and
operational priorities of UNOOSA. Hence statement 3 is correct.
• Committee, Policy and Legal Affairs Section (CPLA) provide substantive, secretariat,
organisational and administrative support.
• Space Applications Section (SAS) plans and implements the United Nations
Programme on Space Applications.
• United Nations Platform for Space-Based Information for Disaster Management and
Emergency Response (UN-SPIDER) is a programme of UNOOSA to leverage space data
and applications for disaster risk reduction.
• The Executive Secretariat of the International Committee on GNSS (ICG) brings
together all global navigation satellite system (GNSS) providers.
• The Office of the Director (OD) oversees and coordinates the strategic direction and
operational priorities of UNOOSA.
Source: https://timesofindia.indiatimes.com/nri/other-news/aarti-holla-maini-with-punjab-
roots-appointed-director-of-un-office-for-outer-space-
affairs/articleshow/101319070.cms?from=mdr
.
IASBABA’S AIPTS 2024 – TEST 4 (CURRENT AFFAIRS – JUNE 2023) SOLUTIONS

Q.73) Solution (b)

Statement Analysis:

• Tadoba-Andhari Tiger Reserve is located in Chandrapur district in Maharashtra.


• It is Maharashtra’s oldest and largest national park.
• The reserve is the second Tiger Reserve in Maharashtra.
• It has a tropical dry deciduous forest with dense woodlands.
• It comprises about 67% of the protected area.
• Teak is the predominant tree species.
• The other trees are Bamboo, Ain, Bija, Dhaudab, Haldu, Salai, Semal, Shisham, Sisoo,
Surya, and Sirus.
Hence option b is correct.

Note: The first Tiger Reserve established in the Maharashtra is Melghat Tiger Reserve
Source: https://www.thehindubusinessline.com/news/national/maharashtras-tadoba-
andhari-tiger-reserve-new-thermal-drones-to-help-detect-forest-fires-intensify-
surveillance/article67011214.ece

Q.74) Solution (a)

Statement Analysis:

• Greedflation simply means (corporate) greed is fuelling inflation.


• It is the phenomenon where corporate greed becomes a significant factor in fueling
inflation. Hence statement 1 is correct.
• It implies that companies exploited the inflation that people were experiencing
by putting up their prices way beyond just covering their increased costs. This was
then used to maximize their profit margins.
• The Indian corporate sector has generated superlative profits in the post-pandemic
period. Hence statement 2 is incorrect.
• Profits during recent times have been nearly thrice the profits corporates earned
earlier.
Source: https://indianexpress.com/article/explained/explained-economics/explainspeaking-
greedflation-inflation-india-explained-8687194/

Q.75) Solution (b)

Statement Analysis:

Colour Alert

1. Yellow Be Aware
.
IASBABA’S AIPTS 2024 – TEST 4 (CURRENT AFFAIRS – JUNE 2023) SOLUTIONS

2. Orange Be Prepared

3. Red Take Action

4. Green All is Well

The Indian Meteorological Department (IMD) uses four colour codes:


• Green (All is well): No advisory is issued.
• Yellow (Be Aware): Yellow indicates severely bad weather spanning several days.
• Orange/Amber (Be prepared): The orange alert is issued as a warning of extremely
bad weather with the potential of disruption in commute with road and rail closures,
and interruption of power supply.
• Red (Take Action): When extremely bad weather conditions are certainly going to
disrupt travel and power and have significant risks to life, the red alert is issued.
Hence option b is correct.
Source: https://newsonair.gov.in/News?title=Met-Department-issues-orange-alert-for-rain-
in-various-parts-of-country-including-Uttarakhand%2c-Himachal-Pradesh%2c-East-
Rajasthan-and-West-Uttar-Pradesh&id=463221

Q.76) Solution (c)

Statement Analysis:

• Tenzing Norgay National Adventure Awards are given every year.


• The award is named after Tenzing Norgay.
• Tenzing Norgay is one of the first two individuals to reach the summit of Mount
Everest along with Edmund Hillary in 1953. Hence statement 1 is correct.
• It was first awarded in 1994.
• It is given by the Ministry of Youth Affairs and Sports. Hence statement 2 is correct.
• Its objective is to recognize the achievements of the person in the field of adventure,
to encourage young people to develop the spirit of endurance, risk-taking,
cooperative teamwork, and quick, ready, and effective reflexes in challenging
situations, and to provide incentives to young people for getting exposed to
adventure activities. Hence statement 3 is correct.
• A person should have excellent performance and outstanding qualities
of leadership, a sense of adventure discipline, and continuous achievement in one
particular field of adventure Land, Air, or Water (Sea).
• The award may be given posthumously if such an occasion arises.
• No award will be conferred for a second time to the same person under the same
category.
• The government of India may cancel or annul the award to any person under
circumstances, which, in the opinion of the Government, might render such person
unworthy of the award.
.
IASBABA’S AIPTS 2024 – TEST 4 (CURRENT AFFAIRS – JUNE 2023) SOLUTIONS

• The award consists of a Bronze statuette, a certificate, a blazer with a silken tie/a
saree, and an award money of Rs. 15 Lakhs.
Source: https://newsonair.gov.in/News?title=Ministry-of-Youth-Affairs-and-Sports-invites-
nominations-for-Tenzing-Norgay-National-Adventure-Award-TANNA-2022&id=463209

Q.77) Solution (b)

Statement Analysis:

Recently, Egypt’s ‘Order of the Nile’ was conferred to India’s Prime Minister, Shri Narendra
Modi.
• The Order of the Nile is Egypt’s highest state honour instituted in 1915. Hence
statement 1 is incorrect.
• It is a pure gold collar consisting of three-square gold units comprising pharaonic
symbols. Hence statement 2 is correct.
• Pharaonic symbols were numerous symbols in the life of ancient Egyptians and varied
in their symbols, rituals, and use.
• It is conferred upon Heads of state, Crown Princes, and Vice-Presidents who offer
Egypt or humanity invaluable services. Hence statement 3 is correct.
Source: https://indianexpress.com/article/india/pm-modi-order-of-the-nile-all-state-
honours-list-8685506/lite/

Q.78) Solution (a)

Statement Analysis:

• The Global Environment Facility (GEF) was initially set up as a fund under World Bank
in 1991. Hence statement 1 is correct.
• In 1992 at the Rio Earth Summit, the GEF was restructured and moved out of the
World Bank system to become a permanent, separate institution.
• Since 1994, however, the World Bank has served as the Trustee of the GEF Trust Fund.
• It serves as a financial mechanism for Convention on Biological Diversity (CBD), United
Nations Framework Convention on Climate Change (UNFCCC), UN Convention to
Combat Desertification (UNCCD), Stockholm Convention on Persistent Organic
Pollutants (POPs), Minamata Convention on Mercury, and Montreal Protocol on
Substances that Deplete the Ozone Layer. Hence statement 2 is correct.
• India is a founder member of GEF. Hence statement 3 is incorrect.
• It is both a donor and recipient of GEF funds.
Source: https://www.downtoearth.org.in/news/wildlife-biodiversity/failure-to-find-
adequate-funds-may-hamper-kunming-montreal-biodiversity-targets-90259

Q.79) Solution (b)


.
IASBABA’S AIPTS 2024 – TEST 4 (CURRENT AFFAIRS – JUNE 2023) SOLUTIONS

Statement Analysis:

• Rani Durgavati was born in 1524 in the Chandela dynasty. Hence statement 1 is
incorrect.
• She is hailed as a patriotic ruler and defender of culture, becoming a symbol of pride
and honour.
• Madan Mahal is a suburban area of Jabalpur famous for the historical Durgavati fort.
The area also has a railway station named Madan Mahal. The fort dates back to 11th
century AD. The fort is well associated with Rani Durgavati the Gond Queen and her
son Veer Narayan. Rani Durgavati eventually died fighting the Mughals, and is hailed
as a martyr in Indian history. Hence statement 2 is correct.
• The University of Jabalpur was renamed Rani Durgavati Vishwavidyalaya by the
Madhya Pradesh government in 1983. Hence statement 3 is correct.
• On June 24, 1988, the Indian government released a postage stamp in memory of her.
• The Durgavati Express (11449/11450) runs between Jabalpur Junction and Jammutawi
and is named for the Queen.
• The third Inshore Patrol Vessel (IPV) of its kind, ICGS Rani Durgavati, was
commissioned by the Indian Coast Guard on July 14, 2018.
Source: https://www.thehindu.com/news/national/other-states/pm-home-minister-to-
attend-events-honouring-martyred-gondwana-queen-in-madhya-
pradesh/article66993995.ece

Q.80) Solution (a)

Statement Analysis:

• A flash flood is a rapid flooding of geomorphic low-lying areas: washes, rivers, dry lakes
and basins. Hence statement 1 is correct.
• It may be caused by heavy rain associated with a severe thunderstorm, hurricane,
tropical storm, or meltwater from ice or snow flowing over ice sheets or snowfields.
• According to NDRF Report, 40 million hectares i.e., 10% of the land mass in India is
prone to floods. Hence statement 2 is incorrect. (70% is too much, isn’t it?)
• On average every year, 5 million hectares of land are affected, 1600 lives are lost and
more than Rs. 1,800 crores is incurred.
• Between 1970 and 2004, 3 floods occurred per year on average. However, between
2005 and 2019, the yearly average rose to 11. 19 districts were affected annually on
an average until 2005. After 2005, the number jumped to 55.
• 2017 analysis suggests that 4.48 million Indians are exposed to riverine floods, the
highest in the world.
Source: https://indianexpress.com/article/explained/everyday-explainers/flashfloods-
landslides-meaning-explained-8686778/
.
IASBABA’S AIPTS 2024 – TEST 4 (CURRENT AFFAIRS – JUNE 2023) SOLUTIONS

Q.81) Solution (c)

Statement Analysis:

• Smart City Mission is a joint effort of the Ministry of Housing and Urban Affairs
(MoHUA), and all state and union territories (UT) governments. Hence statement 1 is
correct.
• It is a Centrally Sponsored Scheme (CSS).
• The Central Government gives financial support to the Mission to the extent of Rs.
48,000 crores over five years i.e. an average of 100 crores per city per year.
• Its fundamental principles include the ‘community at the core’ and ‘more from less’.
Hence statement 2 is correct.

• Its implementation at the city level will be done by a Special Purpose Vehicle (SPV).
Hence statement 3 is correct.
• A special purpose vehicle also called a special purpose entity (SPE), is a subsidiary
created by a parent company to isolate financial risk.
• The SPV is promoted by the state/Union Territory and the urban local body, with
a 50% equity shareholding each.
• The SPV formed as a limited company is governed by the Companies Act, of 2013.
Source: https://newsonair.gov.in/News?title=Union-Minister-Hardeep-Singh-Puri-hails-
Smart-City-Mission%2c-says-over-5800-completed-projects-transformed-urban-
spaces&id=463151

Q.82) Solution (c)

Statement Analysis:

• The Mahadayi River is also known as the Mandovi River.


• It originates in the Western Ghats from the Bhimgad Wildlife Sanctuary in the
Belgaum district of Karnataka. Hence statement 1 is correct.
• It flows for about 81 km before emptying into the Arabian Sea.
• The river is formed by the confluence of two rivers: the Daddi and the Markandeya.
• Its left bank tributaries are the Daddi River, Malaprabha River, and Markandeya River.
• Its right bank tributaries are the Tambaraparani River, Bainganga River, Wardha River.
Hence statement 2 is incorrect.
• Dams on the Mahadayi River are:
.
IASBABA’S AIPTS 2024 – TEST 4 (CURRENT AFFAIRS – JUNE 2023) SOLUTIONS

✓ The Hidkal Dam: Located in the Belagavi district of Karnataka.


✓ The Hidkal Dam: Located in the Belagavi district of Karnataka.
✓ The Selaulim Dam: Located in South Goa.
✓ The Virdi Dam: Located in the Belagavi district of Karnataka.
• The Salim Ali Bird Sanctuary is situated on the island of Chorao, which is located in the
Mandovi River. Hence statement 3 is correct.
• The Mahadayi River is of great importance to the states of Goa and Karnataka, serving
as a source of water for drinking, irrigation, and tourism.
Source: https://www.thehindu.com/news/national/karnataka/kalasa-banduri-project-
tenders-floated-minutes-before-karnataka-poll-announcement-may-not-have-any-
takers/article66989043.ece

Q.83) Solution (d)

Statement Analysis:

• Kedarnath Temple is a Hindu temple dedicated to Lord Shiva.


• It is located on the Garhwal Himalayan range near the Mandakini River, in
the Rudraprayag district of Uttarakhand. Hence statement 1 is correct.
• Mandakini River is a tributary of the Alaknanda River. It emerges from the Chorabari
Glacier. It runs between the Rudraprayag and Sonprayag areas.
• Kedarnath forms one of the four sites of the Chota Char Dham Pilgrimage circuit.
• Chota Char Dham Pilgrimage refers to the journey up the mountains to four sacred
temples in the North Indian state of Uttarakhand.
• The four temples are Yamunotri Dham, Gangotri Dham, Badrinath Dham and
Kedarnath Dham. Hence statement 2 is correct.
• It is one of the 12 Jyotirlingas of Lord Shiva in India.
• Jyotirlinga is a shrine where Lord Shiva is worshipped in the form of a Jyotirlingam.
Hence statement 3 is correct.
.
IASBABA’S AIPTS 2024 – TEST 4 (CURRENT AFFAIRS – JUNE 2023) SOLUTIONS

• It is said that the Pandavas constructed the Kedarnath Temple.


• It was later reconstructed by Adi Shankaracharya in the 8th century A.D.
• It has exquisite architecture and is built of extremely large but evenly shaped grey
stone slabs.
• The stone slabs are interlocked with each other with the use of iron clamps.
• No mortar has been used in the construction of the temple.
• There is a conical rock structure inside the Kedarnath temple that is worshipped as
the Sadashiva form of Shiva.
Source: https://www.indiatoday.in/india/story/high-level-panel-to-probe-alleged-scam-in-
gold-plating-at-kedarnath-temple-2397369-2023-06-24

Q.84) Solution (c)

Statement Analysis:

• The Debrigarh wildlife sanctuary is located in the Bargarh district in the Indian state of
Odisha, covering a total area of 346.91 km2. It is situated near the city of Sambalpur's
Hirakud Dam. Hence statement 1 is incorrect.
• It was declared a wildlife sanctuary in 1985.
• It has dry deciduous forests.
• Hirakud Dam is on the Mahanadi River. Hence statement 2 is correct.
.
IASBABA’S AIPTS 2024 – TEST 4 (CURRENT AFFAIRS – JUNE 2023) SOLUTIONS

Note: Major Protected Areas in Odisha

National Parks:
• Bhitarkanika National Park
• Simlipal National Park-Simlipal is the largest national park in Odisha and is also
considered as one of the principal tiger projects in India. Hence statement 3 is correct

Wildlife Sanctuaries:
• Badrama WLS
• Chilika (Nalaban island) WLS
• Hadgarh WLS
• Baisipalli WLS
• Kotagarh WLS
• Nandankanan WLS
• Lakhari Valley WLS
• Gahirmatha (Marine) WLS
Source: https://www.newindianexpress.com/states/odisha/2023/jun/25/debrigarh-wildlife-
sanctuary-of-odisharecords-higher-prey-density-2588380.html

Q.85) Solution (b)

Statement Analysis:

• The Florence Nightingale Award is the highest national distinction a nurse can achieve
for selfless devotion and exceptional professionalism. Hence statement 1 is incorrect.
• Florence Nightingale was an English social reformer, statistician and the founder of
modern nursing. She came to prominence while serving as a manager and trainer of
nurses during the Crimean War. She organized to care for wounded soldiers at
Constantinople.
• It is given by the Ministry of Health and Family Welfare. Hence statement 2 is correct.
• It was instituted as a mark of recognition for the meritorious services rendered by
nurses and nursing professionals to society.
• The award consists of a cash award of 50000/-, a certificate and a medal. Hence
statement 3 is correct.
• The award is given to outstanding Nursing personnel employed in Central, State/UTs,
Private, Missionary and Voluntary Organizations.
Source: https://www.newindianexpress.com/nation/2023/jun/23/president-confers-
florence-nightingale-awards-on-30-nurses-and-auxiliary-midwives-2587741.html
.
IASBABA’S AIPTS 2024 – TEST 4 (CURRENT AFFAIRS – JUNE 2023) SOLUTIONS

Q.86) Solution (a)

Statement Analysis:

• The emergency provisions are contained in Part XVIII of the Constitution of India,
from Articles 352 to 360. Hence option a is correct.
• These provisions enable the Central government to meet any abnormal situation
effectively.
• The Constitution stipulates three types of emergencies:
✓ National Emergency
✓ Constitutional Emergency
✓ Financial Emergency
Source: https://newsonair.gov.in/News?title=Prime-Minister-Modi-terms-Emergency-as-a-
dark-period%3b-Pays-homage-to-individuals-who-resisted%2c-worked-towards-
strengthening-democracy-in-the-country&id=463148

Q.87) Solution (b)

Statement Analysis:

• Artemis Accord is a non-binding set of principles designed to guide civil space


exploration and use in the 21st century. Hence statement 1 is incorrect.
• It is a non-binding agreement with no financial commitments.
• It was established in 2020 by NASA, in coordination with the U.S. Department of State,
and established the Artemis Accords with eight founder nations.
• Its founding members are Australia, Canada, Italy, Japan, Luxembourg, United Arab
Emirates, the United Kingdom, and the US. Hence statement 2 is correct.
• Artemis Accords signatories as of May 2023 are Australia, Bahrain, Brazil, Canada,
Colombia, Czech Republic, France, Israel, Italy, Japan, Luxembourg, Mexico, New
Zealand, Nigeria, Poland, the Republic of Korea, Romania, Rwanda, Saudi Arabia,
Singapore, Spain, Ukraine, the United Arab Emirates, the United Kingdom, and the
United States.
Source: https://newsonair.gov.in/News?title=India-and-US-sign-Artemis-Accord-to-take-
Space-cooperation-to-new-heights%3b-ISRO-NASA-to-launch-joint-space-mission-next-
year&id=463087

Q.88) Solution (c)

Statement Analysis:

• The Food Corporation of India is a statutory body under the Food Corporation Act of
1964. Hence statement 1 is correct.
• It works under the Ministry of Consumer Affairs, Food, and Public Distribution. Hence
statement 2 is correct.
.
IASBABA’S AIPTS 2024 – TEST 4 (CURRENT AFFAIRS – JUNE 2023) SOLUTIONS

• It is headquartered in New Delhi.

Objectives of FCI:
✓ Effective price support operations for safeguarding the interests of the farmers.
✓ Distribution of food grains throughout the country for the public distribution system.
✓ Maintaining a satisfactory level of operational and buffer stocks of foodgrains
to ensure National Food Security.
✓ Effective Price Support Operations for safeguarding the interest of farmers.
Source: https://www.outlookindia.com/business/first-e-auction-of-fci-wheat-under-omss-
to-be-held-on-june-28-to-offload-3-5-lakh-ton-news-295225

Q.89) Solution (c)

Statement Analysis:

• The National Human Rights Commission (NHRC) is a multi-member body consisting of


a chairperson, five full-time members, and seven deemed members. Hence statement
1 is correct.
• Chairperson will be a retired chief justice of India or a judge of the Supreme Court.
• The chairman and members are appointed by the President on the recommendations
of a six-member committee consisting of:-
✓ Prime Minister (head)
✓ Speaker of the Lok Sabha
✓ Deputy Chairman of the Rajya Sabha
✓ Leaders of the Opposition in both Houses of Parliament
✓ Union Home Minister.
• They serve for three years or until they attain the age of 70 years, whichever is earlier.
Hence statement 2 is incorrect.
• The President can remove them from office under specific circumstances.
• They can be removed only on the charges of proven misbehavior or incapacity if
proved by an inquiry conducted by a Supreme Court Judge. Hence statement 3 is
correct.
Source: https://newsonair.gov.in/News?title=NHRC-issues-advisory-to-Centre%2c-
States%2c-and-UTs-to-mitigate-deliberate-self-harm-and-suicide-attempts-by-
prisoners&id=463090

Q.90) Solution (b)

Statement Analysis:

• It is a specialised agency of the United Nations (UN).


o UN: is an international organization founded in 1945.
.
IASBABA’S AIPTS 2024 – TEST 4 (CURRENT AFFAIRS – JUNE 2023) SOLUTIONS

o It works for maintaining international peace and security, protect human


rights, deliver humanitarian aid, promote sustainable development and
uphold international law.
• Objective: promoting world peace and security through international cooperation in
education, the sciences, and culture.
• HQ: Paris, France.
• Members:-
o It has 194 member states and 12 associate members, as well as partners in the
nongovernmental, intergovernmental, and private sectors.
o Recently USA rejoined UNESCO. Hence statement 3 is not correct.
o UN member states not UNESCO members: Israel, Liechtenstein.

Issue of USA Leaving UNESCO

• In 2011, UNESCO inducted Palestine as a member.


• This led to the US halting the agency’s funding, worth millions of dollars, under then-
President Barack Obama.
• Reason to stop funding UNESCO:-
o Israel and the United States termed the inclusion of Palestine and UNESCO’s
naming of what Israel claims were ancient Jewish sites as Palestinian heritage
sites as examples of anti–Israel bias.
o US laws, owing to the country’s historical ties with Israel, prohibit funding to
any UN agency that implies recognition of the Palestinians’ demands for their
own state.
• Consequently, in 2019 the US and Israel pulled out of UNESCO citing bias in the
organisation in the Palestine issue.
o The US had pulled out of UNESCO once earlier in 1984 and then rejoined in
2003.
• Recent Developments: it was negotiated recently through an agreement in 2022 that
the USA will begin giving UNESCO funds again.
• Issue of Palestine:-
o The Palestinians claim the West Bank, east Jerusalem and Gaza
Strip territories captured by Israel in the 1967 war for an independent state.
o Israel disagrees with Palestine’s claims.
o Palestine is not recognised as a sovereign state by the United Nations.

India and UNESCO

• India has been a founding member of UNESCO. Hence statement 1 is correct


• It had ratified UNESCO’s Constitution in 1946, while still under colonial rule.
• India has been continuously re-elected to the UNESCO Executive Board since 1946.
.
IASBABA’S AIPTS 2024 – TEST 4 (CURRENT AFFAIRS – JUNE 2023) SOLUTIONS

• Recently, India won the re-election to the executive board of the UN’s cultural and
education organisation for the 2021-25 term. Hence statement 2 is correct.

Source: https://indianexpress.com/article/explained/us-rejoin-unesco-israel-palestine-
explained-8661069/

Q.91) Solution (d)

Statement Analysis:

• The Bureau of Indian Standards (BIS) was established in 1986.


• It was originally enacted under the Bureau of Indian Standards Act of 1986.
• A new act (to replace the BIS Act of 1986) was introduced in the Lok Sabha, which was
subsequently passed by both houses of Parliament in 2016.
• It is headquartered in New Delhi.
• It works under the Ministry of Consumer Affairs, Food & Public Distribution.
• It has regional offices in Kolkata (Eastern), Chennai (Southern), Mumbai (Western),
Chandigarh (Northern), and Delhi (Central).
• Functions of BIS:
✓ Standards formulation
✓ Product Certification
✓ Hallmarking
✓ Laboratory services
✓ Training services – National Institute of Training for Standardization
✓ Consumer Affairs and Publicity
Hence option d is correct.
Source: https://newsonair.gov.in/News?title=BIS-publishes-Food-Serving-Utensils-with-an-
aim-to-reduce-plastic-pollution-and-promoting-sustainability&id=463037

Q.92) Solution (c)

Statement Analysis:

The Directorate of Civil Aviation (DGCA) is the regulatory body in the field of Civil
Aviation primarily dealing with safety issues.
The functions of the DGCA are:
• Registration of civil aircraft.
• Licensing of pilots, aircraft maintenance engineers, and flight engineers. Hence
statement 1 is correct.
• Formulation of standards of airworthiness for civil aircraft registered in India. Hence
statement 2 is correct.
• It conducts examinations and checks for that purpose.
.
IASBABA’S AIPTS 2024 – TEST 4 (CURRENT AFFAIRS – JUNE 2023) SOLUTIONS

• Licensing of air traffic controllers.


• Conducting an investigation into accidents/incidents and taking accident preventive
measures.
• Coordination at the national level for flexible use of air space by civil and military air
traffic agencies.
• Interaction with ICAO for the provision of more air routes for civil use through Indian
air space.
• Promoting indigenous design and manufacture of aircraft.

Note:
• Air traffic controllers are responsible for ensuring the safe and efficient movement of
aircraft in the airspace.
• International Civil Aviation Organization(ICAO) is a specialized agency of the United
Nations that coordinates the principles and techniques of international air navigation.
Source: https://newsonair.gov.in/News?title=DGCA-suspends-licence-of-Air-India-pilot-for-
allowing-unauthorised-person-into-cockpit&id=463032

Q.93) Solution (a)

Statement Analysis:

• The Global Gender Gap Index is published annually since 2006 by the World Economic
Forum (WEF). Hence option a is correct.
• It benchmarks the current state and evolution of gender parity across four key
dimensions:
✓ Economic Participation and Opportunity
✓ Educational Attainment
✓ Health and Survival
✓ Political Empowerment
• On each of these four sub-indices as well as on the overall index the GGG index
provides scores between 0 and 1.
✓ 1: full gender parity
✓ 0: complete imparity
• It is the longest-standing index tracking the progress of numerous countries’ efforts
toward closing these gaps over time since its inception.

Note: WEF is a non-profit organization based in Geneva, Switzerland. It engages the foremost
political, business, cultural, and other leaders of society to shape global, regional, and industry
agendas.
Source: https://www.downtoearth.org.in/news/africa/sub-saharan-africa-might-need-
nearly-a-century-to-achieve-gender-parity-global-gender-gap-report-90202
.
IASBABA’S AIPTS 2024 – TEST 4 (CURRENT AFFAIRS – JUNE 2023) SOLUTIONS

Q.94) Solution (d)

Statement Analysis:

• The Canary Islands -a Spanish archipelago off the coast of northwestern Africa. It is
part of Spain. It is about 1300 km South of mainland Spain and 115 km West of the
African coast (Morocco). Hence option d is correct.
• It is located in the Atlantic Ocean. It is an archipelago.
• These Islands were formed by volcanic eruptions millions of years ago.

Source: https://newsonair.gov.in/News?title=More-than-30-migrants-feared-dead-after-
small-boat-headed-for-Spain%26%2339%3bs-Canary-Islands-sank&id=462985

Q.95) Solution (c)

Statement Analysis:

Indian Mesolithic sites:


• Mirzapur district of Uttar Pradesh.
• Tilwara, Bagor, and Ganeshwar in Rajasthan
• Patne, Pachad, and Hatkhamba in Maharashtra
• Pachmarhi, Bhimbetka, Baghor II, Baghor III, and Ghagharia in Madhya Pradesh
• Paisra in Bihar
• Kuchai in Odisha
• Birbhanpur in West Bengal
• Muchatla Chintamanu Gavi, Guntur, and Gauri Gundam in Andhra Pradesh
• Sanganakallu in Karnataka
• Tenmalai in Kerala
Hence option c is correct.
Source: https://www.thehindu.com/news/national/andhra-pradesh/archaeologist-reports-
mesolithic-era-rock-paintings-in-guntur/article66986465.ece/amp/

Q.96) Solution (c)


.
IASBABA’S AIPTS 2024 – TEST 4 (CURRENT AFFAIRS – JUNE 2023) SOLUTIONS

Statement Analysis:

The characteristics of Mesolithic Rock Paintings:


• This period mainly saw the use of red colour. Though Hence statement 1 is correct.
• It has a larger variety of themes but the paintings are smaller in size.
• The hunting scenes were predominant in the period. Hence statement 2 is correct.
• The paintings depict people hunting in groups and hunting armed with barbed spears,
pointed sticks, arrows, and bows.
• Animals depicted include elephants, bison, tiger, boar, deer, antelope, leopard,
panther, rhinoceros, fish, frog, lizard, squirrel, and at times birds. Hence statement 3
is correct.
• Animals were painted in a naturalistic style and humans were depicted in a stylistic
manner.
• The young, old, children, and women equally find a place in these paintings.
• Women are painted both nude and clothed.
• In many of the rock shelters handprints, fist prints, and dots made by the fingertips
are found.
• One of the most prominent examples of the Mesolithic rock painting is found
in Bhimbetka, Madhya Pradesh.
Source: https://www.thehindu.com/news/national/andhra-pradesh/archaeologist-reports-
mesolithic-era-rock-paintings-in-guntur/article66986465.ece/amp/

Q.97) Solution (a)

Statement Analysis:

• GEMCOVAC-OM is India’s first mRNA-based Omicron-specific booster vaccine. Hence


statement 1 is correct.
• It is developed by Gennova Biopharmaceuticals Ltd. It is supported under the Mission
COVID Suraksha.
• It is a thermo-stable vaccine, which does not require ultra-cold chain
infrastructure used for other approved mRNA-based vaccines. This makes it easy for
deployment all across India.
• It is delivered intra-dermally using a needle-free injection device system.
• It is available for adults over the age of 18.
• Two doses will be administered intramuscularly 28 days apart.
• It works by introducing a piece of mRNA that corresponds to a viral protein.
• It is created in the nucleus when DNA is transcribed by RNA polymerase to create pre-
mRNA. Hence statement 2 is incorrect.
• mRNA is a single-stranded RNA molecule that complements DNA.
.
IASBABA’S AIPTS 2024 – TEST 4 (CURRENT AFFAIRS – JUNE 2023) SOLUTIONS

Note: Intradermal injection is a shallow or superficial injection of a substance into the dermis
(located between the epidermis and the hypodermis). When administered intradermal in
participants as a booster, it generates significantly higher immune responses.
Source: https://indianexpress.com/article/health-wellness/indias-first-mrna-based-omicron-
specific-booster-vaccine-approved-8675607/

Q.98) Solution (d)

Statement Analysis:

• The 69th Constitutional Amendment Act of 1991 conferred the UT of Delhi with
a special status. Hence option d is correct.
• After India attained independence on August 15, 1947, New Delhi was declared
the capital of the Union of India.
• The Union Territory of Delhi was constituted from the Chief Commissioner’s Province
of Delhi under the States Reorganization Act of 1956.
• It re-designated Delhi as the National Capital Territory of Delhi.
• It designated the administrator of Delhi (under Article 239) as the Lieutenant
Governor (LG) of Delhi.
• These special provisions for Delhi were provided under Article 239AA.
Source: https://indianexpress.com/article/upsc-current-affairs/the-360-upsc-debate-delhi-
statehood-debate-reasons-for-and-against-8678120/

Q.99) Solution (b)

Statement Analysis:

• The Statue of Unity was built in honor of Sardar Vallabhbhai Patel. Hence statement
1 is incorrect.
• It was inaugurated on 31st October 2018. It marked the 143rd birth anniversary of
Sardar Patel.
• The Statue of Unity is the tallest statue in the world. It stands at 182-metre (600 feet
approx.)
• It is taller than China’s Spring Temple Buddha statue and almost double the height
of the Statue of Liberty (93 meters tall) in the United States. Hence statement 2 is
correct.
• In 2020, it was added to the ‘Eight Wonders’ of the Shanghai Cooperation
Organization (SCO).
• It is located in Sadhu Bet Island, Gujarat. The island is on the Narmada River. Hence
statement 3 is correct.
• It is designed by Padma Bhushan recipient sculptor Ram V Sutar.
• The intricate bronze cladding work was done by a Chinese foundry, the Jiangxi Toqine
Company (JTQ).
.
IASBABA’S AIPTS 2024 – TEST 4 (CURRENT AFFAIRS – JUNE 2023) SOLUTIONS

Source: https://indianexpress.com/article/cities/ahmedabad/tender-issued-to-start-
suspended-helicopter-joyride-at-statue-of-unity-8676142/

Q.100) Solution (a)

Statement Analysis:

• The Gangotri National Park (GNP) is located in Uttarkashi District, Uttarakhand. Hence
option a is correct.
• It is located in the upper catchment of Bhagirathi River.
• The northeastern section of the park forms the international boundary of India and
Tibet (China).
• It borders Kedarnath Wildlife Sanctuary and Govind National Park.
• The mountains in the park are part of the Gangotri Group of the Garhwal Himalayas.
• The park harbors Western Himalayan Subalpine Conifer Forests at lower elevations
and Western Himalayan Alpine Shrub and Meadows at higher elevations.
• Vegetation consists of chirpine deodar, fir, spruce, oak and rhododendrons.

Source: https://timesofindia.indiatimes.com/city/dehradun/gangotri-park-to-give-50-ha-to-
army-itbp-for-bunkers/articleshow/101122022.cms?from=mdr

You might also like